Aviation Weather Flashcards

1
Q

The Mid-Level Significant weather (SIGWX) Chart is used to determine an overview of flying weather conditions between what altitudes?
A. 10,000 feet AGL and 45,000 feet AGL.
B. 10,000 feet MSL and FL 450.
C. 10,000 feet AGL and FL 450

A

B. 10,000 feet MSL and FL 450.
Answer (B) is correct. (AC 00-45H Chap 5) ( ? )
Dispatchers and flight crew members can use the Mid-Level Significant Weather (SIGWX) Chart to determine an overview of selected flying weather conditions. The altitude areas for this chart are between 10,000 feet MSL and FL 450. This information can be used for flight planning and weather briefings before departure and during flight.

How well did you know this?
1
Not at all
2
3
4
5
Perfectly
2
Q

Maximum turbulence potential charts (GTG-3) are issued
A. every morning at 0400.
B. hourly.
C. two times a day.

A

B. hourly.
Answer (B) is correct. (AC 00-45H Chap 5) ( ? )
Maximum turbulence potential charts (GTG-3) graphics are computer generated, four-dimensional forecasts of information related to the likelihood of encountering Clear Air Turbulence (CAT) associated with upper-level fronts and jet streams. This forecast is updated every hour and includes 06, 09, and 12-hour forecasts, which are updated every 3 hours, starting at 00Z.

How well did you know this?
1
Not at all
2
3
4
5
Perfectly
3
Q

A cyclone is
A. a hurricane force storm in the Indian Ocean with highest sustained winds of 64 knots or higher.
B. a tropical depression in the Northwest Pacific with sustained winds of 63 knots.
C. a tropical storm in the Atlantic with highest sustained winds of 35 through 64 knots.

A

A. a hurricane force storm in the Indian Ocean with highest sustained winds of 64 knots or higher.
Answer (A) is correct. (AC 00-6B Chap 21) ( ? )
Tropical cyclone is a general term for any low that originates over tropical oceans. Tropical cyclones are classified according to their intensity based on average 1-minute wind speeds. Tropical cyclone international classifications are
1.
Tropical Depression–highest sustained winds up to 33 knots;
2.
Tropical Storm–highest sustained winds of 34 through 63 knots; and
3.
Hurricane, Cyclone, or Typhoon–highest sustained winds 64 knots or more.
Strong tropical cyclones are known by different names in different regions of the world. A tropical cyclone in the Atlantic and eastern Pacific is a “hurricane”; in the western Pacific, “typhoon”; near Australia and in the Indian Ocean, simply “cyclone.”

How well did you know this?
1
Not at all
2
3
4
5
Perfectly
4
Q

METAR KHRO 131753Z 09007KT 7SM FEW020 BKN040CB 30/27 A3001.
SPECI KHRO 131815Z 13017G26KT 3SM +TSRA SCT020 BKN045TCU 29/24 A2983 RMK RAB12 WS TKO LDG RW14R FRQ LTGICCG VC.

What change has taken place between 1753 and 1815 UTC at Harrison (KHRO)?

A. The ceiling lowered and cumulonimbus clouds developed.
B. Visibility reduced to IFR conditions.
C. Thundershowers began at 12 minutes past the hour.

A

Thundershowers began at 12 minutes past the hour.
Answer (C) is correct. (AC 00-45H Chap 3) ( ? )
The special report (SPECI) taken at 1815 UTC at KHRO reports thunderstorm with heavy rain (+TSRA), and the remarks state that the rain began at 12 min. past the hour (RAB12).

How well did you know this?
1
Not at all
2
3
4
5
Perfectly
5
Q
Determine the approximate wind direction and velocity at FL 240 over the station in central Oklahoma.
A.	280° at 30 knots.
B.	100° at 30 knots.
C.	280° at 35 knots.
See Figure 153,154,155
A
Answer (A) is correct. (AC 00-45H Chap 5) ( ? )
To determine the approximate wind direction, you must interpolate between values for FL 180 (500-mb chart) and FL 300 (300-mb chart). The wind direction and speed are determined by the flag at the reporting station. First decode the two given pressure levels:
FL 180 (500 mb)
=
280°
at
25 kt.
FL 300 (300 mb)
=
280°
at
35 kt.
Difference
=
0°
at
10 kt.
Interpolation for each value gives the following:
FL 240 = 280° at 30 kt.
How well did you know this?
1
Not at all
2
3
4
5
Perfectly
6
Q

Which weather phenomenon signals the beginning of the mature stage of a thunderstorm?
A. The start of rain at the surface.
B. Growth rate of the cloud is at its maximum.
C. The appearance of an anvil top.

A

A. The start of rain at the surface.
Answer (A) is correct. (AC 00-6B Chap 19) ( ? )
The mature stage of a thunderstorm begins when rain begins falling at the surface. This means that downdrafts have developed sufficiently to carry water all the way through the thunderstorm.

How well did you know this?
1
Not at all
2
3
4
5
Perfectly
7
Q

Cumuliform clouds signify
A. unstable air conditions.
B. low temperatures.
C. mountainous terrain below.

A

A. unstable air conditions.
Answer (A) is correct. (AC 00-6B Chap 13) ( ? )
Unstable air favors convection. Cumulus (meaning heap) clouds form in a convective updraft and build upward. Within an unstable layer, clouds are cumuliform. The vertical extent of the cloud depends on the depth of the unstable layer.

How well did you know this?
1
Not at all
2
3
4
5
Perfectly
8
Q

Middle clouds are considered to be the clouds between
A. 1,000 and 6,500 feet AGL.
B. 20,000 feet AGL and above.
C. 6,500 and 20,000 feet AGL.

A

C. 6,500 and 20,000 feet AGL.
Answer (C) is correct. (FAA-H-8083-25B Chap 12) ( ? )
The middle cloud family consists of the altostratus, altocumulus, and nimbostratus clouds. These clouds are primarily water, much of which may be supercooled. The height of the bases of these clouds ranges from about 6,500 to 20,000 feet in middle latitudes.

How well did you know this?
1
Not at all
2
3
4
5
Perfectly
9
Q

Steady-state thunderstorms are usually associated with
A. the mature stage.
B. weather systems.
C. surface heating.

A

B. weather systems.
Answer (B) is correct. (FAA-H-8083-25B Chap 12) ( ? )
Weather systems such as fronts, converging winds, and troughs aloft force upward motion spawning these storms that often form into squall lines. Steady-state thunderstorms usually are associated with weather systems.

How well did you know this?
1
Not at all
2
3
4
5
Perfectly
10
Q

What type turbulence should be reported when it momentarily causes slight, erratic changes in altitude and/or attitude, one-third to two-thirds of the time?
A. Intermittent light turbulence.
B. Occasional light chop.
C. Moderate chop.

A

A. Intermittent light turbulence.
Answer (A) is correct. (AIM Para 7-1-25) ( ? )
Light turbulence is defined as turbulence that momentarily causes slight, erratic changes in altitude and/or attitude. Intermittent is defined as occurring from one-third to two-thirds of the time.

How well did you know this?
1
Not at all
2
3
4
5
Perfectly
11
Q
What was the local Central Standard Time of the Aviation Routine Weather Report at Austin (KAUS)?
A.	5:53 p.m.
B.	11:53 a.m.
C.	10:53 p.m.
See figure 145
A

B. 11:53 a.m.
Answer (B) is correct. (AC 00-45H Chap 3) ( ? )
The date/time after Austin (KAUS) on Fig. 145 is 131753Z. The first two digits are the date and the last four digits are the time, followed by Z to denote coordinated universal time (UTC). Thus, the report was issued at 1753 UTC. To determine Central Standard Time, you must subtract 6 hr. from UTC; thus, the time is 11:53 a.m. (1753 – 6 = 1153) CST.

How well did you know this?
1
Not at all
2
3
4
5
Perfectly
12
Q

What will be the wind and temperature trend for a DEN-ICT-OKC flight at 11,000 feet?
A. Windspeed increase slightly.
B. Wind shift from calm to a westerly direction.
C. Temperature decrease.
See figure 149

A

A. Windspeed increase slightly.
Answer (A) is correct. (AC 00-45H Chap 7) ( ? )
To determine the wind and temperature trend, the winds and temperatures aloft must be interpolated for 11,000 ft. The table below shows data for 9,000 and 12,000 ft. and the interpolated data for 11,000 ft.

9,000
12,000
11,000
DEN
9900+09
9900+04
9900+06
ICT
0607+08
9900+04
0602+05
OKC
1106+10
9900+05
1102+07
The best answer is that wind speed increases slightly from light and variable in DEN to 2 kt. at ICT and OKC.
How well did you know this?
1
Not at all
2
3
4
5
Perfectly
13
Q

Gusts in tropical cyclones can exceed the average one-minute wind speed by as much as
A. 100%
B. 50%
C. 75%

A

B. 50%
Answer (B) is correct. (AC 00-6B Chap 21) ( ? )
Tropical cyclones are classified according to their intensity based on average one-minute wind speeds. Wind gusts in these storms may be as much as 50% higher than the average one-minute wind speeds.

How well did you know this?
1
Not at all
2
3
4
5
Perfectly
14
Q

Tropical storms form when
A. a system has peak wind gusts that do not exceed 65 knots.
B. a system of air surrounds an area with pressure of 48 mb.
C. a system has sustained winds of 34 to 63 knots.

A

C. a system has sustained winds of 34 to 63 knots.
Answer (C) is correct. (AC 00-6B Chap 21) ( ? )
The classification of a tropical storm is based upon sustained winds of 34 through 63 knots.

How well did you know this?
1
Not at all
2
3
4
5
Perfectly
15
Q

Updrafts in the mature stage of a thunderstorm can exceed
A. 6,000 feet per minute.
B. 8,000 feet per minute.
C. 7,000 feet per minute.

A

A. 6,000 feet per minute.
Answer (A) is correct. (AC 00-6A Chap 11) ( ? )
Updrafts and downdrafts in close proximity create strong vertical shear and a very turbulent environment. Updrafts reach a maximum with speeds possibly exceeding 6,000 feet per minute.

How well did you know this?
1
Not at all
2
3
4
5
Perfectly
16
Q

Which type frontal system is normally crossed by the jetstream?
A. Occluded front.
B. Warm front.
C. Cold front and warm front.

A

A. Occluded front.
Answer (A) is correct. (AC 00-6B Chap 10) ( ? )
The development of a surface low usually is south of the jet stream and moves nearer as the low deepens. The occluding low moves north of the jet stream, and the jet stream crosses the frontal system near the point of occlusion.

How well did you know this?
1
Not at all
2
3
4
5
Perfectly
17
Q

The peak wind at KAMA was reported to be from 320° true at 39 knots,
A. with gusts to 43 knots.
B. which occurred at 1743Z.
C. with .43 of an inch liquid precipitation since the last report.
See figure 145

A

B. which occurred at 1743Z.
Answer (B) is correct. (AC 00-45H Chap 3) ( ? )
Whenever the peak wind exceeds 25 kt., PK WND will be included in the remarks with three digits for direction and two or three digits for speed followed by the time (in hours and minutes or just minutes past the hour) of occurrence. The time of the METAR report for KAMA is 1755Z, and the remark PK WND 32039/43 means the peak wind 320° true at 39 kt. occurred at 43 min. past the hour, or at 1743Z.

How well did you know this?
1
Not at all
2
3
4
5
Perfectly
18
Q

Shear turbulence may be expected
A. no more than 10 miles from a severe thunderstorm.
B. usually within 20 miles of a severe thunderstorm.
C. probably not more than 15 miles from a severe thunderstorm.

A

B. usually within 20 miles of a severe thunderstorm.
Answer (B) is correct. (AC 00-24C) ( ? )
Outside of the cloud, shear turbulence has been encountered several thousand feet above and 20 miles laterally from a severe storm.

How well did you know this?
1
Not at all
2
3
4
5
Perfectly
19
Q

Weather conditions expected to occur in the vicinity of the airport, but not at the airport, are denoted by the letters “VC.” When VC appears in a Terminal Aerodrome Forecast, it covers a geographical area of
A. a 5-to-10 statute mile radius from the airport.
B. a 5-mile radius of the center of a runway complex.
C. 10 miles of the station originating the forecast.

A

A. a 5-to-10 statute mile radius from the airport.
Answer (A) is correct. (AC 00-45H Chap 7) ( ? )
When VC appears in a TAF, it applies to weather conditions expected to occur in an area within a 5- to 10-SM radius of the airport, but not at the airport itself.

How well did you know this?
1
Not at all
2
3
4
5
Perfectly
20
Q

What type weather system is approaching the Oregon Coast from the northwest?
A. Cold front.
B. HIGH.
C. LOW.

A

C. LOW.
Answer (C) is correct. (AC 00-45H Chap 5) ( ? )
A three-dimensional picture can be developed by following the change in height of the height contours. These are the solid lines on Fig. 153, and the height is indicated on the lines. The contour that is approaching the Oregon coast is lower than the contours over the northwestern U.S. Thus, a low-pressure area must be approaching.

How well did you know this?
1
Not at all
2
3
4
5
Perfectly
21
Q

A calm wind that is forecast, in the International Terminal Aerodrome Forecast (TAF), is encoded as
A. 00003KT.
B. 00000KT.
C. VRB00KT.

A

B. 00000KT.
Answer (B) is correct. (AC 00-45H Chap 7) ( ? )
In the International Terminal Aerodrome Forecast (TAF), a calm wind that is forecast is encoded as 00000KT.

How well did you know this?
1
Not at all
2
3
4
5
Perfectly
22
Q

Convective SIGMETs are issued for a line of thunderstorms at least
A. 100 miles long with severe thunderstorms affecting 50 percent of its length.
B. 60 miles long with thunderstorms affecting 40 percent of its length.
C. 40 miles long with thunderstorms affecting 50 percent of its length.

A

B. 60 miles long with thunderstorms affecting 40 percent of its length.
Answer (B) is correct. (AC 00-45H) ( ? )
A convective SIGMET will be issued when (1) a line of thunderstorms is at least 60 miles long with thunderstorms affecting at least 40% of its length or (2) an area of active thunderstorms is (a) affecting at least 3,000 square miles, (b) covering at least 40% of the area concerned, and (c) exhibiting a very strong radar reflectivity intensity or a significant satellite or lightning signature.

How well did you know this?
1
Not at all
2
3
4
5
Perfectly
23
Q

As you approach an airport to land, you observe a convective cloud over the airport with virga below it. This could indicate
A. the presence of a microburst.
B. smooth air.
C. heavy rain showers.

A

A. the presence of a microburst.
Answer (A) is correct. (FAA-H-8083-25B Chap 12) ( ? )
Rain that falls through the atmosphere but evaporates prior to striking the ground is known as virga. The process of evaporation cools the air around the virga and can create strong downdrafts and in some cases microbursts.

How well did you know this?
1
Not at all
2
3
4
5
Perfectly
24
Q
If involved in a microburst encounter, in which aircraft positions will the most severe downdraft occur?
A.	3 and 4.
B.	2 and 3.
C.	4 and 5.
See figure 144
A

A. 3 and 4.
Answer (A) is correct. (AC 00-54) ( ? )
The most severe downdrafts occur at the center of the microburst, indicated by positions 3 and 4 in Fig. 144.

How well did you know this?
1
Not at all
2
3
4
5
Perfectly
25
Q

What is the approximate rate unsaturated air will cool flowing upslope?
A. 4°C per 1,000 feet.
B. 2°C per 1,000 feet.
C. 3°C per 1,000 feet.

A

C. 3°C per 1,000 feet.
Answer (C) is correct. (AC 00-6B Chap 11) ( ? )
Unsaturated air moving upward and downward cools and warms at about 3°C (5.4°F) per 1,000 ft. This is the dry adiabatic rate of temperature change.

How well did you know this?
1
Not at all
2
3
4
5
Perfectly
26
Q

The Federal Aviation Administration’s Flight Information Service Data Link (FISDL) is designed to provide data on a common frequency to flight crews from
A. 5,000 feet MSL to 17,500 feet MSL.
B. 17,500 feet MSL down to 5,000 feet AGL.
C. 17,500 feet AGL down to 5,000 feet MSL.

A

B. 17,500 feet MSL down to 5,000 feet AGL.
Answer (B) is correct. (AC 00-63A) ( ? )
The FAA’s Flight Information Service Data Link (FISDL) system is designed to provide coverage throughout the continental U.S. from 5,000 ft. AGL to 17,500 ft. MSL, except in those areas where this is unfeasible due to mountainous terrain.

How well did you know this?
1
Not at all
2
3
4
5
Perfectly
27
Q

A public severe thunderstorm watch implies
A. 58 mph winds or greater and/or surface hail of 1 inch or more in diameter.
B. 50 knots or greater and/or surface hail of 1/2 inch or greater.
C. 45 mph winds or greater and/or surface hail of 1 inch or more in diameter.

A

A. 58 mph winds or greater and/or surface hail of 1 inch or more in diameter.
Answer (A) is correct. (AC 00-45H Chap 5) ( ? )
The National Weather Service may issue a public severe thunderstorm watch when conditions are favorable to winds of 58 mph (50 kt.) or greater and/or surface hail of 1 inch or more in diameter.

How well did you know this?
1
Not at all
2
3
4
5
Perfectly
28
Q

What weather database can a dispatcher access concerning wind shear activity at an airport?
A. ATIS.
B. TWIP.
C. AWOS.

A

B. TWIP.
Answer (B) is correct. (AIM Para 7-1-26) ( ? )
Dispatchers can access the TWIP database and send messages to specific aircraft whenever wind shear activity begins or ends at an airport.

How well did you know this?
1
Not at all
2
3
4
5
Perfectly
29
Q

What is indicated by the following report?
TYR UUA/OV TYR180015/TM 1757/FL310/TP B737/TB MOD-SEV CAT 350-390

A. A special METAR issued on the 18th day of the month at 1757Z.
B. An urgent pilot report for moderate to severe clear air turbulence.
C. A routine pilot report for overcast conditions from flight levels 350-390.

A

B. An urgent pilot report for moderate to severe clear air turbulence.
Answer (B) is correct. (AC 00-45H Sect 3.2) ( ? )
The UUA found in the first section of the report indicates an “Urgent Upper Air” report. The /TB MOD-SEV CAT 350-390 in the last section of the report indicates moderate to severe clear air turbulence for flight levels 350 to 390.

How well did you know this?
1
Not at all
2
3
4
5
Perfectly
30
Q

METAR KFSO 031053Z VRB02KT 7SM MIFG SKC 15/14 A3012 RMK SLP993 60000 T01500139 56012
In the above METAR, the SLP993 60000 indicates

A. sea-level pressure 999.3 hectopascals which in the last 6 hours has dropped .4 hectopascals.
B. sea-level pressure 999.3 hectopascals and in the last 6 hours that four-tenths of an inch of precipitation has fallen.
C. sea-level pressure 999.3 hectopascals and a trace amount of precipitation has occurred over the last 3 hours.

A

C. sea-level pressure 999.3 hectopascals and a trace amount of precipitation has occurred over the last 3 hours.
Answer (C) is correct. (AC 00-45H Chap 3) ( ? )
In the remarks (RMK) section of the METAR, the coded group SLP993 means the sea-level pressure (SLP) is 999.3 hectopascals. A coded element beginning with the number 6 indicates the 3- and 6-hr. precipitation amount. The “6” is followed by four digits indicating precipitation about using tens, units, tenths, and hundredths of an inch. When a trace amount of precipitation has occurred, four zeros will follow the number 6 (60000).

How well did you know this?
1
Not at all
2
3
4
5
Perfectly
31
Q

What wind direction and speed aloft are forecast by this WINDS AND TEMPERATURE ALOFT FORECAST (FB) report for FL 390 - “731960”?
A. 230° at 119 knots.
B. 073° at 196 knots.
C. 131° at 96 knots.

A

A. 230° at 119 knots.
Answer (A) is correct. (AC 00-45H Chap 7) ( ? )
The first two digits are the direction group, and since it is greater than 36, the wind speed must be 100 kt. or greater. Subtract 50 from the direction group to determine wind direction of 230° (73 – 50 = 23). The second two digits are the wind speed; since it was determined that the wind was 100 kt. or greater, add 100 to this group. Thus, the wind speed is 119 kt. (100 + 19 = 119).

How well did you know this?
1
Not at all
2
3
4
5
Perfectly
32
Q

Precipitation-induced fog
A. results from relatively warmer rain or drizzle falling through cooler air.
B. results from relatively cooler rain or drizzle falling through warmer air.
C. is usually of short duration.

A

A. results from relatively warmer rain or drizzle falling through cooler air.
Answer (A) is correct. (AC 00-6B Chap 16) ( ? )
Precipitation-induced fog results from relatively warmer rain or drizzle falling through cooler air. This fog may extend over large areas, completely suspending air operations.

How well did you know this?
1
Not at all
2
3
4
5
Perfectly
33
Q

The horizontal wind shear, critical for turbulence (moderate or greater) per 150 miles is
A. not a factor, only vertical shear is a factor.
B. 18 knots or less.
C. greater than 18 knots.

A

C. greater than 18 knots.
Answer (C) is correct. (AC 00-30C) ( ? )
The horizontal wind shear critical for turbulence (moderate or greater) is greater than 18 kt. per 150 NM.

How well did you know this?
1
Not at all
2
3
4
5
Perfectly
34
Q

Cumulus clouds often indicate
A. a temperature inversion.
B. a dry adiabatic lapse rate.
C. possible turbulence.

A

C. possible turbulence.
Answer (C) is correct. (AC 00-6B) ( ? )
Cumulus clouds are formed in a convective updraft, build upward, and are associated with turbulence.

How well did you know this?
1
Not at all
2
3
4
5
Perfectly
35
Q

Except for temperature, ice fog forms in conditions favorable to
A. Arctic fog.
B. advection fog.
C. radiation fog.

A

C. radiation fog.
Answer (C) is correct. (FAA-H-8083-25B Chap 12) ( ? )
Conditions favorable for this type of fog are the same as for Arctic fog except for the temperature. Radiation fog forms at night under clear skies with calm winds when heat absorbed by the Earth’s surface during the day is radiated into space. Provided a deep enough layer of moist air is present near the ground, as the Earth’s surface continues to cool, the humidity will reach 100% and fog will form. Radiation fog varies in depth from 3 feet to about 1,000 feet, is always found at ground level, and usually remains stationary. This type of fog can reduce visibility to near zero at times and makes flying very hazardous during takeoffs and landings.

How well did you know this?
1
Not at all
2
3
4
5
Perfectly
36
Q

A severe thunderstorm is one in which the surface wind is
A. 50 knots or greater and/or surface hail is 1/2 inch or more in diameter.
B. 58 mph or greater and/or surface hail is 3/4 inch or more in diameter.
C. 45 knots or greater and/or surface hail is 1 inch or more in diameter.

A

B. 58 mph or greater and/or surface hail is 3/4 inch or more in diameter.
Answer (B) is correct. (AC 00-45H Chap 3) ( ? )
A severe thunderstorm is one in which the surface winds are greater than or equal to 58 mph (50 kt.), hail at the surface is greater than or equal to 3/4 in. in diameter, or tornadoes are present.

How well did you know this?
1
Not at all
2
3
4
5
Perfectly
37
Q

Storm gust fronts often move as much as
A. 20 miles ahead of the associated precipitation.
B. 15 miles ahead of the associated precipitation.
C. 10 miles ahead of the associated precipitation.

A

B. 15 miles ahead of the associated precipitation.
Answer (B) is correct. (AC 00-24C) ( ? )
Gust fronts often move far ahead (up to 15 miles) of associated precipitation. The gust front causes a rapid and sometimes drastic change in surface wind ahead of an approaching storm.

How well did you know this?
1
Not at all
2
3
4
5
Perfectly
38
Q

The prevailing visibility in the following METAR is
METAR KFSM 131756Z AUTO 00000KT M1/4SM R25/0600V1000FT -RA FG VV004 06/05 A2989 RMK A02 $.

A. a mean (average) of 1/4 statute mile.
B. less than 1/4 statute mile.
C. measured 1/4 statute mile.

A

B. less than 1/4 statute mile.
Answer (B) is correct. (AC 00-45H Chap 3) ( ? )
Prevailing visibility is reported in statute miles with a space and then fractions of statute miles, as needed, with SM appended to it. This METAR was produced at an automated station, as indicated by the modifier AUTO. At an automated station, visibility of less than 1/4 SM is reported as M1/4SM.

How well did you know this?
1
Not at all
2
3
4
5
Perfectly
39
Q

A prognostic chart depicts the conditions
A. forecast to exist at a specific time in the future.
B. existing at the surface during the past 6 hours.
C. which presently exist from the 1,000-millibar through the 700-millibar level.

A

A. forecast to exist at a specific time in the future.
Answer (A) is correct. (AC 00-45H Chap 8) ( ? )
Prognostic charts show conditions as they are forecast to be at the valid time (UTC or Zulu) for the chart. The charts are issued four times daily.

How well did you know this?
1
Not at all
2
3
4
5
Perfectly
40
Q

A station is forecasting wind and temperature aloft to be 280° at 205 knots; temperature –51°C at FL 390. How would this data be encoded in the FB?
A. 789951
B. 780051
C. 280051

A

A. 789951
Answer (A) is correct. (AC 00-45H Chap 7) ( ? )
At FL 390, a 280° wind at 205 kt. is encoded as 789951. The first two digits are the direction. The second two digits are velocity. When wind speed is forecast at 200 kt. or greater, the wind group is coded as 99, and 50 is added to the direction code. Here the direction is 78 for 280°. The last two digits indicate the temperature, and minus signs are omitted above 24,000 ft. MSL.

How well did you know this?
1
Not at all
2
3
4
5
Perfectly
41
Q

What characterizes a ground-based inversion?
A. Convection currents at the surface.
B. Poor visibility.
C. Cold temperatures.

A

B. Poor visibility.
Answer (B) is correct. (AC 00-6B Chap 21) ( ? )
A ground-based inversion is characterized by poor visibility as a result of trapping fog, smoke, and other restrictions into low levels of the atmosphere.

How well did you know this?
1
Not at all
2
3
4
5
Perfectly
42
Q

If the ambient temperature is colder than standard at FL 310, what is the relationship between true altitude and pressure altitude?
A. True altitude is lower than 31,000 feet.
B. Pressure altitude is lower than true altitude.
C. They are both the same, 31,000 feet.

A

A. True altitude is lower than 31,000 feet.
Answer (A) is correct. (AC 00-6B Chap 5) ( ? )
At FL 310, the altimeter is set to pressure altitude (29.92). If the outside air temperature (ambient) is colder than standard, the air is compressed and heavier in weight per unit volume than on a warm day. Thus, in colder-than-standard air, the true altitude is lower than the pressure altitude.

How well did you know this?
1
Not at all
2
3
4
5
Perfectly
43
Q

SPECI KGLS 131802Z 10012G21KT 060V140 2SM +SHRA SCT005 BKN035 OVC050CB 24/23 A2980 RMK RAB57 WS TKO RW09L WSHFT 58 FROPA.
This SPECI report at Galveston (KGLS) indicates which condition?

A. Precipitation started at 57 after the hour.
B. 5,000 feet overcast with towering cumulus.
C. Wind steady at 100° magnetic at 12 knots, gusts to 21.

A

A. Precipitation started at 57 after the hour.
Answer (A) is correct. (AC 00-45H Chap 3) ( ? )
The remarks (RMK) section of the SPECI report at KGLS indicates rain began at 57 min. past the hour (RAB57); wind shear occurred during takeoff on RWY 09L (WS TKO RW09L); and the wind shifted in direction at 58 min. past the hour due to a frontal passage (WSHFT 58 FROPA).

How well did you know this?
1
Not at all
2
3
4
5
Perfectly
44
Q

What wind direction and speed aloft are forecast by this WINDS AND TEMPERATURE ALOFT FORECAST (FB) for FL 390 - “750649”?
A. 150° at 6 knots.
B. 250° at 106 knots.
C. 350° at 64 knots.

A

B. 250° at 106 knots.
Answer (B) is correct. (AC 00-45H Chap 5) ( ? )
Coded directions with wind speed greater than 100 kt. range from 51 to 86. Fifty must be subtracted from the direction group and 100 added to the speed group. In this forecast, the wind direction is 250° (75 – 50 = 25), and the wind speed is 106 kt. (100 + 06 = 106).

How well did you know this?
1
Not at all
2
3
4
5
Perfectly
45
Q

A low-pressure area is an area of
A. stagnant air.
B. ascending air.
C. descending air.

A

B. ascending air.
Answer (B) is correct. (FAA-H-8083-25B Chap 12) ( ? )
Air flows into low pressure areas to replace rising air.

How well did you know this?
1
Not at all
2
3
4
5
Perfectly
46
Q

What type conditions can be expected for a flight scheduled to land at Austin-Bergstrom International Airport (KAUS) at 1200Z?
A. IFR conditions due to low visibility.
B. Chance of 1 statute mile visibility and cumulonimbus clouds.
C. MVFR conditions due to low ceilings.
See Figure 147

A

C. MVFR conditions due to low ceilings.
Answer (C) is correct. (AC 00-45H Chap 7) ( ? )
IFR conditions are due to low visibility in rain and mist, not low ceilings and fog. At KAUS from 0900 to 1800Z (UTC), the forecast is wind 180° at 7 kt., visibility greater than 6 SM, and a broken ceiling at 1,500 ft. AGL. Thus, the expected conditions are MVFR conditions due to a low ceiling that is below 3,000 ft. AGL

How well did you know this?
1
Not at all
2
3
4
5
Perfectly
47
Q

What weather feature occurs at altitude levels near the tropopause?
A. Thin layers of cirrus (ice crystal) clouds at the tropopause level.
B. Maximum winds and narrow wind shear zones.
C. Abrupt temperature increase above the tropopause.

A

B. Maximum winds and narrow wind shear zones.
Answer (B) is correct. (AC 00-30C) ( ? )
Temperature and wind vary greatly in the vicinity of the tropopause. Maximum winds generally occur near the tropopause. These winds create narrow zones of wind shear, which often generate hazardous turbulence.

How well did you know this?
1
Not at all
2
3
4
5
Perfectly
48
Q

Which type jet stream can be expected to cause the greater turbulence?
A. A jet stream associated with a wide isotherm spacing.
B. A straight jet stream associated with a high pressure ridge.
C. A curving jet stream associated with a deep low pressure trough.

A

C. A curving jet stream associated with a deep low pressure trough.
Answer (C) is correct. (AC 00-30C) ( ? )
The jet stream cirrus shield is its association with turbulence. Extensive cirrus cloudiness often occurs with deepening surface and upper lows. These deepening systems produce the greatest turbulence.

How well did you know this?
1
Not at all
2
3
4
5
Perfectly
49
Q

Which weather condition is an example of a nonfrontal instability band?
A. Squall line.
B. Advective fog.
C. Frontogenesis.

A

A. Squall line.
Answer (A) is correct. (FAA-H-8083-25B Chap 12) ( ? )
An instability line is a narrow, nonfrontal line or band of convective activity. If this activity is fully developed thunderstorms, it is called a squall line.

How well did you know this?
1
Not at all
2
3
4
5
Perfectly
50
Q

The three jet streams are
A. the polar jetstream, the Tropic of Cancer jetstream, and the Maritime jetstream.
B. the polar front jetstream, the subtropical jetstream, and the polar night jetstream.
C. the polar front jetstream, the subtropical jetstream, and the polar jetstream.

A

B. the polar front jetstream, the subtropical jetstream, and the polar night jetstream.
Answer (B) is correct. (AC 00-6B Chap 8) ( ? )
There are three jetstreams. The polar front jetstream is associated with the polar front or the division between the cold polar and warm tropical air masses. The subtropical jetstream is a very persistent circumpolar jetstream found on the northern periphery of the tropical latitudes between 20° and 30° north latitude. The polar night jetstream is found in the stratosphere in the vicinity of the Arctic Circle during the winter months.

How well did you know this?
1
Not at all
2
3
4
5
Perfectly
51
Q

Maximum downdrafts in a microburst encounter may be as strong as
A. 7,000 ft/min.
B. 6,000 ft/min.
C. 8,000 ft/min.

A

B. 6,000 ft/min.
Answer (B) is correct. (AIM Para 7-1-26) ( ? )
Downdrafts in a microburst can be as strong as 6,000 ft/min. Horizontal winds near the surface can be as strong as 45 kt., resulting in a 90-kt. wind shear. The strong horizontal winds occur within a few hundred feet of the ground.

How well did you know this?
1
Not at all
2
3
4
5
Perfectly
52
Q

Convective clouds which penetrate a stratus layer can produce which threat to instrument flight?
A. Embedded thunderstorms.
B. Clear air turbulence.
C. Freezing rain.

A

A. Embedded thunderstorms.
Answer (A) is correct. (FAA-H-8083-25B Chap 12) ( ? )
A stratus layer may sometimes form in a mildly stable layer while convective clouds penetrate the layer and possibly form thunderstorms. These thunderstorms may be almost or entirely embedded in a massive stratus layer and pose an unseen threat to instrument flight.

How well did you know this?
1
Not at all
2
3
4
5
Perfectly
53
Q

Which wind-shear condition results in an increase in airspeed?
A. Decreasing tailwind and increasing headwind.
B. Increasing tailwind and decreasing headwind.
C. Increasing tailwind and headwind.

A

A. Decreasing tailwind and increasing headwind.
Answer (A) is correct. (AC 00-54) ( ? )
When a tailwind shears to a headwind, the airspeed initially increases, the aircraft pitches up, and the altitude increases.

How well did you know this?
1
Not at all
2
3
4
5
Perfectly
54
Q
What is the forecast temperature at ATL for the 3,000-foot level?
A.	Not reported.
B.	+6°C.
C.	+6°F.
See figure 149
A

A. Not reported.
Answer (A) is correct. (AC 00-45H Chap 5) ( ? )
In the Winds and Temperature Aloft Forecasts, no temperatures are forecast for the 3,000-ft. level or for a level within 2,500 ft. of station elevation.

How well did you know this?
1
Not at all
2
3
4
5
Perfectly
55
Q

Under what conditions would clear air turbulence (CAT) most likely be encountered?
A. When constant pressure charts show 60-knot isotachs less than 20 NM apart.
B. When a sharp trough is moving at a speed less than 20 knots.
C. When constant pressure charts show 20-knot isotachs less than 60 NM apart.

A

C. When constant pressure charts show 20-knot isotachs less than 60 NM apart.
Answer (C) is correct. (AC 00-6B Chap 6) ( ? )
When constant pressure charts show 20-kt. isotachs less than 60 NM apart, there is sufficient horizontal shear for CAT. These conditions normally occur on the polar side of the jet stream.

How well did you know this?
1
Not at all
2
3
4
5
Perfectly
56
Q

SIGMETs (other than domestic convective SIGMETs) may be valid for not more than
A. 6 hours for other phenomena.
B. 4 hours for other phenomena.
C. 4 hours for other phenomena and may be issued 2 hours before the valid time.

A

B. 4 hours for other phenomena.
Answer (B) is correct. (AC 00-45H Chap 5) ( ? )
SIGMETs are issued for 6 hour periods for conditions associated with hurricanes and 4 hours for all other events. If conditions persist beyond the forecast period, the SIGMET is updated and reissued.

How well did you know this?
1
Not at all
2
3
4
5
Perfectly
57
Q

What weather improvement was reported at Lubbock (KLBB) between 1750 and 1818 UTC?
A. The vertical visibility improved by 2,000 feet.
B. The temperature and dew point spread improved.
C. The wind shift and frontal passage at 1812Z.
See figure 145

A

A. The vertical visibility improved by 2,000 feet.
Answer (A) is correct. (AC 00-45H Chap 3) ( ? )
The vertical visibility at KLBB at 1750 UTC was reported to be 1,000 ft. (VV010). At 1818 UTC, the vertical visibility was reported to be 3,000 ft. (VV030), an improvement of 2,000 ft. from the 1750 UTC report.

How well did you know this?
1
Not at all
2
3
4
5
Perfectly
58
Q

What is the relative moisture content of the air mass along the California coast?
A. Dry in the southern part of the state, with increased chance for wet conditions in the mid portion, then drying over the northern section.
B. Very wet with high potential for condensation.
C. Moist enough for condensation along the length of the coast.
See figure 153,154,155

A

A. Dry in the southern part of the state, with increased chance for wet conditions in the mid portion, then drying over the northern section.
Answer (A) is correct. (AC 00-45H Chap 5) ( ? )
Determine the moisture content by comparing the temperature/dew point spread of the stations along the California coast at the 200-, 300-, and 500-mb pressure levels. The spread, if given, is located at the lower left of the station symbol. Both the 500- and 200-mb charts indicate relatively wide spread dry conditions. The 300-mb chart shows that moisture is high in northern California

How well did you know this?
1
Not at all
2
3
4
5
Perfectly
59
Q

The decrease in wind speed from the core of the jet stream is the greatest on
A. the acute angle side of the jet stream.
B. the equatorial side of the jet stream.
C. the polar side of the jet stream.

A

C. the polar side of the jet stream.
Answer (C) is correct. (AC 00-30C) ( ? )
The rate of decreased wind speed is considerably greater on the polar side than on the equatorial side. The magnitude of wind shear is greater on the polar side than on the equatorial side.

How well did you know this?
1
Not at all
2
3
4
5
Perfectly
60
Q

Maximum downdrafts in a microburst encounter may be as strong as
A. 1,500 ft/min.
B. 6,000 ft/min.
C. 4,500 ft/min.

A

B. 6,000 ft/min.
Answer (B) is correct. (AIM Para 7-1-26) ( ? )
Downdrafts in a microburst can be as strong as 6,000 ft/min. Horizontal winds near the surface can be as strong as 45 kt., resulting in a 90-kt. wind shear. The strong horizontal winds occur within a few hundred feet of the ground.

How well did you know this?
1
Not at all
2
3
4
5
Perfectly
61
Q

Which pressure is defined as station pressure?
A. Altimeter setting.
B. Actual pressure at field elevation.
C. Station barometric pressure reduced to sea level.

A

B. Actual pressure at field elevation.
Answer (B) is correct. (AC 00-6B Chap 5) ( ? )
Pressure can be measured only at the point of measurement. The pressure measured at a station or airport is “station pressure,” or the actual pressure at field elevation.

How well did you know this?
1
Not at all
2
3
4
5
Perfectly
62
Q

METAR KFSO 031053Z VRB02KT 7SM MIFG SKC 15/14 A3012 RMK SLP993 SNINCR 1/10
In the above METAR, the SNINCR 1/10 indicates

A. sea-level pressure 999.3 hectopascals and 1” of snow in the last 10 hours.
B. sea-level pressure 999.3 hectopascals and a snow depth of 10” and a 1” increase in the last hour.
C. sea-level pressure 999.3 hectopascals and 0.10” of snow on the runway landing zone during the last hour.

A

B. sea-level pressure 999.3 hectopascals and a snow depth of 10” and a 1” increase in the last hour.
Answer (B) is correct. (AC 00-45H Chap 3) ( ? )
SNINCR (Snow Increasing Rapidly) indicates a snow depth increase of 1 inch in the past hour with a total depth on the ground of 10 inches.

How well did you know this?
1
Not at all
2
3
4
5
Perfectly
63
Q

What weather condition occurs at the altitude where the dewpoint lapse rate and the dry adiabatic lapse rate converge?
A. Stable air changes to unstable air.
B. Precipitation starts.
C. Cloud bases form.

A

C. Cloud bases form.
Answer (C) is correct. (AC 00-6B Chap 11) ( ? )
The altitude where the dew point lapse rate and the dry adiabatic lapse rate converge is where convective cloud bases form. Unsaturated air in a convective current cools at about 3.0°C (5.4°F) per 1,000 ft.; dew point decreases at about 5/9°C (1°F) per 1,000 ft. When they converge, cloud bases form.

How well did you know this?
1
Not at all
2
3
4
5
Perfectly
64
Q

What term describes an elongated area of low pressure?
A. Trough.
B. Ridge.
C. Hurricane or typhoon.

A

A. Trough.
Answer (A) is correct. (AC 00-6B Chap 5) ( ? )
A trough is defined as an elongated area of low pressure with the lowest pressure along a line marking maximum cyclonic curvature.

How well did you know this?
1
Not at all
2
3
4
5
Perfectly
65
Q

A SIGMET may be issued
A. for an area where conditions are deemed to have a significant effect on the safety of aircraft operations.
B. for an area where conditions are likely to develop air turbulence above FL 310.
C. when expectations for dust storms or icing is predicted to be higher than 40% over a 1,000 square mile area.

A

A. for an area where conditions are deemed to have a significant effect on the safety of aircraft operations.
Answer (A) is correct. (AC 00-45H Chap 5) ( ? )
A SIGMET may be issued when any of the specific conditions occur or are expected to occur in an area deemed to have a significant effect on the safety of aircraft operations.

How well did you know this?
1
Not at all
2
3
4
5
Perfectly
66
Q

Land surface cooling on the coast results in
A. a land breeze.
B. a sea breeze.
C. a chinook wind.

A

A. a land breeze.
Answer (A) is correct. (AC 00-6B Chap 9) ( ? )
At night, the wind blows from cool land to warmer water and creates a land breeze.

How well did you know this?
1
Not at all
2
3
4
5
Perfectly
67
Q

What determines how icing is reported on a PIREP?
A. Rate of accumulation.
B. Type of ice.
C. Thickness of ice.

A

A. Rate of accumulation.
Answer (A) is correct. (AIM Para 7-1-21) ( ? )
Icing intensities are classified based on the rate of icing accumulation.

How well did you know this?
1
Not at all
2
3
4
5
Perfectly
68
Q

En route at FL 270, the altimeter is set correctly. On descent, a pilot fails to set the local altimeter setting of 30.57. If the field elevation is 650 feet, and the altimeter is functioning properly, what will it indicate upon landing?
A. 585 feet.
B. 1,300 feet.
C. Sea level.

A

C. Sea level.
Answer (C) is correct. (FAA-H-8083-25B Chap 8) ( ? )
If an altimeter is set to 29.92 for flight at FL 270 and is not adjusted to the correct altimeter setting of 30.57 during descent, the altimeter will indicate an altitude lower than actual by 650 ft. (1,000 ft. × the difference between the two altimeter settings, which is .65). Thus, if the airplane lands at an airport with a field elevation of 650 ft., the altimeter will indicate sea level (650 ft. airport elevation – 650 ft. altimeter setting error).

How well did you know this?
1
Not at all
2
3
4
5
Perfectly
69
Q

Where do squall lines most often develop?
A. In an occluded front.
B. Behind a stationary front.
C. Ahead of a cold front.

A

C. Ahead of a cold front.
Answer (C) is correct. (FAA-H-8083-25B Chap 12) ( ? )
A squall line is a nonfrontal, narrow band of active thunderstorms. Most often squall lines develop ahead of a cold front in moist, unstable air.

How well did you know this?
1
Not at all
2
3
4
5
Perfectly
70
Q

All ATC facilities using radar weather processors with the ability to determine precipitation intensity will describe the intensity as
A. light, moderate, heavy, extreme.
B. light, moderate, heavy, extreme, severe.
C. light, moderate, heavy, intense, extreme, severe.

A

A. light, moderate, heavy, extreme.
Answer (A) is correct. (AIM Para 7-1-13) ( ? )
All ATC facilities using radar weather processors with the ability to determine precipitation intensity will describe the intensity to pilots as light, moderate, heavy, or extreme.

How well did you know this?
1
Not at all
2
3
4
5
Perfectly
71
Q

Which type weather conditions are covered in the Convective SIGMET?
A. Embedded thunderstorms, lines of thunderstorms, and thunderstorms with 3/4-inch hail or tornadoes.
B. Cumulonimbus clouds with tops above the tropopause and thunderstorms with 1/2-inch hail or funnel clouds.
C. Any thunderstorm with a severity level of VIP 2 or more.

A

A. Embedded thunderstorms, lines of thunderstorms, and thunderstorms with 3/4-inch hail or tornadoes.
Answer (A) is correct. (AC 00-45H Chap 3) ( ? )
Convective SIGMETs are issued for (1) severe thunderstorm due to surface winds greater than 50 kt., hail at the surface equal to or greater than 3/4 in. in diameter, or tornadoes; (2) embedded thunderstorms; (3) line of thunderstorms; or (4) thunderstorms greater than or equal to VIP level 4 affecting 40% or more of an area at least 3,000 square mi.

How well did you know this?
1
Not at all
2
3
4
5
Perfectly
72
Q

When encountering severe turbulence, you should
A. Maintain a constant altitude.
B. Slow to turbulent air penetration speed.
C. Maintain constant airspeed and altitude.

A

B. Slow to turbulent air penetration speed.
Answer (B) is correct. (FAA H-8083-25B)
When encountering severe turbulence, you should slow down to turbulent air penetration speed, maneuvering speed (VA). A faster airspeed could cause excessive stress on the aircraft when maneuvering.

How well did you know this?
1
Not at all
2
3
4
5
Perfectly
73
Q

Downdrafts in a thunderstorm may exceed
A. 24 knots.
B. 34 knots.
C. 14 knots.

A

A. 24 knots.
Answer (A) is correct. (AC 00-6A Chap 11) ( ? )
Downdrafts can exceed 2,500 feet per minute (fpm). We convert that to knots by multiplying
2,500 fpm × 60 minutes = 150,000 feet per hour
1 nautical mile = 6,076 feet
150,000 feet per hour ÷ 6,076 feet = 24.69 knots

How well did you know this?
1
Not at all
2
3
4
5
Perfectly
74
Q

To allow pilots of in-trail lighter aircraft to make flight path adjustments to avoid wake turbulence, pilots of heavy and large jet aircraft should fly
A. on the established glidepath and on the approach course centerline or runway centerline extended.
B. above the established glidepath and slightly downwind of the on-course centerline.
C. below the established glidepath and slightly to either side of the on-course centerline.

A

A. on the established glidepath and on the approach course centerline or runway centerline extended.
Answer (A) is correct. (AIM Para 7-3-8) ( ? )
To allow pilots of in-trail lighter aircraft to make flight path adjustments to avoid wake turbulence, pilots of aircraft that produce strong wake vortices, which include heavy and large jet aircraft, should fly on the established glide path and on the approach course centerline or the extended centerline of the runway.

How well did you know this?
1
Not at all
2
3
4
5
Perfectly
75
Q

Which INITIAL flight deck indications should a pilot be aware of when a headwind shears to a calm wind?
A. Indicated airspeed decreases, aircraft pitches down, and altitude decreases.
B. Indicated airspeed decreases, aircraft pitches up, and altitude decreases.
C. Indicated airspeed increases, aircraft pitches down, and altitude increases.

A

A. Indicated airspeed decreases, aircraft pitches down, and altitude decreases.
Answer (A) is correct. (AC 00-54) ( ? )
As the wind shears from a headwind to a calm wind, there is a quick reduction in the headwind component and a decrease in aircraft performance. The indications will be a decrease in indicated airspeed, a pitch down of the airplane’s nose, and a decrease in altitude.

How well did you know this?
1
Not at all
2
3
4
5
Perfectly
76
Q

Which area or areas of the Northern Hemisphere experience a generally east to west movement of weather systems?
A. Subtropical only.
B. Arctic only.
C. Arctic and subtropical.

A

C. Arctic and subtropical.
Answer (C) is correct. (AC 00-6B Chap 10) ( ? )
The high-pressure belt at about 30° north latitude forces air outward at the surface to the north and to the south. The southward moving air is deflected by the Coriolis force, becoming the well-known subtropical northeast (moving northeast to southwest) trade winds.
In the arctic region (60° north latitude and above), the heavier, denser air moves south at a low level toward the Equator but is turned to the right by the Coriolis force, causing the low-level polar easterlies (moving east to west).

How well did you know this?
1
Not at all
2
3
4
5
Perfectly
77
Q
What is the approximate temperature for a flight from southern California to central Kansas at FL 350?
A.	–39°C.
B.	–48°C.
C.	–44°C.
See figure 153,154,155
A

C. –44°C.
Answer (C) is correct. (AC 00-45H Chap 5) ( ? )
You will need to use the 300-mb (FL 390) chart and the 200-mb (FL 300) chart and interpolate for FL 350.
The temperature is located in the upper left corner of the station on a constant pressure chart. On the 300-mb chart, the station in Southern California is reporting –36°C. Following the reporting stations along the intended flight path, you will find temperatures of –36, –38, –39, –42, and –41°C on the 300-mb chart. Similarly, you will find temperatures of –47, –47, –48, –48, and –52°C on the 200-mb chart. When averaging these temperatures at each level, the difference is about 9°, or approximately 1° per thousand feet of altitude. For FL 350, the estimated temperature would be –44°C.

How well did you know this?
1
Not at all
2
3
4
5
Perfectly
78
Q

What is the lowest cloud in the stationary group associated with a mountain wave?
A. Rotor cloud.
B. Low stratus.
C. Standing lenticular.

A

A. Rotor cloud.
Answer (A) is correct. (AC 00-6B Chap 17) ( ? )
When moisture is sufficient to produce clouds on the lee side of a mountain, the lowest clouds associated with mountain wave are rotor clouds. These are found under each crest of the standing wave.

How well did you know this?
1
Not at all
2
3
4
5
Perfectly
79
Q

Which type of icing is associated with the smallest size of water droplet similar to that found in low-level stratus clouds?
A. Rime ice.
B. Clear ice.
C. Frost ice.

A

A. Rime ice.
Answer (A) is correct. (AC 00-6B Chap 18) ( ? )
Rime ice forms when drops are small, such as those found in low-level stratus clouds. Its irregular shape makes it very effective in decreasing aerodynamic efficiency of airfoils.

How well did you know this?
1
Not at all
2
3
4
5
Perfectly
80
Q

An increase in temperature with an altitude increase
A. is indication of an inversion.
B. denotes the beginning of the stratosphere.
C. means a cold front passage.

A

A. is indication of an inversion.
Answer (A) is correct. (FAA-H-8083-25B Chap 12) ( ? )
Normally, as air rises and expands in the atmosphere, the temperature decreases. However, when the temperature of the air rises with altitude, this indicates that a temperature inversion exists.

How well did you know this?
1
Not at all
2
3
4
5
Perfectly
81
Q

What condition produces the most frequent type of ground- or surface-based temperature inversion?
A. Terrestrial radiation on a clear, relatively calm night.
B. Widespread sinking of air within a thick layer aloft resulting in heating by compression.
C. The movement of colder air under warm air or the movement of warm air over cold air.

A

A. Terrestrial radiation on a clear, relatively calm night.
Answer (A) is correct. (AC 00-6B Chap 2) ( ? )
An inversion often develops near the ground on clear, cool nights when the wind is relatively calm. The ground radiates heat and cools much faster than the overlying air. Air in contact with the ground becomes cold, while the temperature a few hundred feet above changes very little.

How well did you know this?
1
Not at all
2
3
4
5
Perfectly
82
Q

Which weather condition is present when the tropical storm is upgraded to a hurricane?
A. Highest windspeed, 100 knots or more.
B. A clear area or hurricane eye has formed.
C. Sustained winds of 64 knots or more.

A

C. Sustained winds of 64 knots or more.
Answer (C) is correct. (AC 00-6B Chap 21) ( ? )
Tropical cyclones are classified by their intensity based on average 1-min. wind speeds. To be classified as a hurricane, the cyclone must have sustained winds of 64 kt. or more.

How well did you know this?
1
Not at all
2
3
4
5
Perfectly
83
Q

What is the primary cause of all changes in the Earth’s weather?
A. Movement of air masses from moist areas to dry areas.
B. Variations of solar energy at the Earth’s surface.
C. Changes in air pressure over the Earth’s surface.

A

B. Variations of solar energy at the Earth’s surface.
Answer (B) is correct. (FAA-H-8083-25B Chap 12) ( ? )
Every physical process of weather is accompanied by or is the result of a heat exchange. Unequal solar heating of the Earth’s surface causes differences in air pressure that result in all changes in the Earth’s weather.

How well did you know this?
1
Not at all
2
3
4
5
Perfectly
84
Q

When you hear a SIGMET on an ATC frequency forecasting severe icing conditions on the route to your destination, you plan for
A. very little airframe icing because of an OAT of -10°C or colder, the moisture is already frozen and cannot adhere to airplane surfaces.
B. the possibility of freezing rain and freezing drizzle that can accumulate on and beyond the limits of any system.
C. the installed transport category airplane ice protection system protecting against all types and levels of icing as designed.

A

B. the possibility of freezing rain and freezing drizzle that can accumulate on and beyond the limits of any system.
Answer (B) is correct. (AIM PC Glossary) ( ? )
Severe icing is defined by the rate of accumulation such that deicing/anti-icing equipment fails to reduce or control the hazard.

How well did you know this?
1
Not at all
2
3
4
5
Perfectly
85
Q

Clear air turbulence associated with a jet stream is
A. most commonly found in temperatures between -40° and -50°C.
B. most commonly found in the vicinity of the tropopause.
C. similar to that associated with a tropical maritime front.

A

B. most commonly found in the vicinity of the tropopause.
Answer (B) is correct. (AC 00-30C) ( ? )
Clear air turbulence (CAT) associated with a jet stream is most often found in the vicinity of the tropopause, which is the boundary layer between the troposphere and the stratosphere.

How well did you know this?
1
Not at all
2
3
4
5
Perfectly
86
Q

With high altitude moisture present, cirrus clouds will form
A. on the polar side of the jet stream.
B. on the acute angle side of the jet stream.
C. on the equatorial side of the jet stream.

A

C. on the equatorial side of the jet stream.
Answer (C) is correct. (AC 00-30C) ( ? )
Air travels in a corkscrew path around the jet core with upward motion on the equatorial side. When high level moisture is available, cirriform clouds form on the equatorial side of the jet stream.

How well did you know this?
1
Not at all
2
3
4
5
Perfectly
87
Q

Steep frontal surfaces are usually associated with
A. fast moving cold front.
B. fast moving warm front.
C. dry lines.

A

A. fast moving cold front.
Answer (A) is correct. (FAA-H-8083-25B Chap 12) ( ? )
A fast moving, steep cold front forces upward motion of the warm air along its leading edge. The frontal slope is steep near the leading edge as cold air replaces warm air.

How well did you know this?
1
Not at all
2
3
4
5
Perfectly
88
Q

Which conditions result in the formation of frost?
A. Dew collects on the surface and then freezes because the surface temperature is lower than the air temperature.
B. The temperature of the collecting surface is at or below freezing and small droplets of moisture are falling.
C. Temperature of the collecting surface is below the dewpoint and the dewpoint is also below freezing.

A

C. Temperature of the collecting surface is below the dewpoint and the dewpoint is also below freezing.
Answer (C) is correct. (FAA-H-8083-25B Chap 12) ( ? )
Frost forms when both the temperature of the collecting surface is below the dew point and the dew point is also below freezing. Frost is the deposition of water vapor to ice crystals.

How well did you know this?
1
Not at all
2
3
4
5
Perfectly
89
Q

METAR KMAF 131756Z 02020KT 12SM BKN025 OVC250 27/18 A3009 RMK RAE44.
Which weather condition is indicated by this METAR report at Midland (KMAF)?

A. Wind was 020° magnetic at 20 knots.
B. The ceiling was at 25,000 feet MSL.
C. Rain of unknown intensity ended 16 minutes before the hour.

A

C. Rain of unknown intensity ended 16 minutes before the hour.
Answer (C) is correct. (AC 00-45H Chap 3) ( ? )
The report at KMAF indicates RMK RAE44, which means remarks follow: rain ended 44 min. past the hour (or 16 min. before the hour).

How well did you know this?
1
Not at all
2
3
4
5
Perfectly
90
Q

Which flight conditions of a large jet airplane create the most severe flight hazard by generating wingtip vortices of the greatest strength?
A. Heavy, slow, gear and flaps down.
B. Heavy, fast, gear and flaps down.
C. Heavy, slow, gear and flaps up.

A

C. Heavy, slow, gear and flaps up.
Answer (C) is correct. (AIM Para 7-3-3) ( ? )
The strength of a vortex is governed by the weight, speed, and shape of the wing of the generating aircraft. The vortex characteristics of any aircraft can be changed by extension of flaps and landing gear, as well as by changes in speed. The greatest vortex strength of a large jet airplane occurs when the airplane is heavy, slow, and clean (gear and flaps up).

How well did you know this?
1
Not at all
2
3
4
5
Perfectly
91
Q

What weather is predicted by the term VCTS in a Terminal Aerodrome Forecast?
A. Thunderstorms may occur over the station and within 50 miles of the station.
B. Thunderstorms are expected in the vicinity.
C. Thunderstorms are expected between 5 and 25 miles of the runway complex.

A

B. Thunderstorms are expected in the vicinity.
Answer (B) is correct. (AC 00-45H Chap 7) ( ? )
The term VC (vicinity) applies to weather conditions expected within a 5- to 10-SM radius of the airport, but not at the airport itself. Thus, VCTS in a TAF means thunderstorms are expected in the vicinity of the airport.

How well did you know this?
1
Not at all
2
3
4
5
Perfectly
92
Q

What feature is associated with a temperature inversion?
A. Air mass thunderstorms.
B. A stable layer of air.
C. An unstable layer of air.

A

B. A stable layer of air.
Answer (B) is correct. (AC 00-6B Chap 2) ( ? )
A temperature inversion is defined as an increase in temperature with height; i.e., the normal lapse rate is inverted. Thus, any warm air rises to its own temperature and forms a stable layer of air.

How well did you know this?
1
Not at all
2
3
4
5
Perfectly
93
Q
What approximate wind direction, speed, and temperature (relative to ISA) are expected for a flight over TUS at FL 270?
A.	010° true; 5 knots; ISA +13°C.
B.	347° magnetic; 5 knots; ISA –10°C.
C.	350° true; 5 knots; ISA +5°C.
See Figure 149
A
A.	010° true; 5 knots; ISA +13°C.
Answer (A) is correct. (AC 00-45H Chap 5) ( ? )
For conditions at FL 270 over TUS in Fig. 149, interpolate between values at FL 240 and FL 300. First decode the two given flight levels:
FL 240
=
050°
at
05 kt.
and
–17°C
FL 300
=
330°
at
05 kt.
and
–33°C
Difference
=
80°
at
0 kt.
and
–16°C
Interpolation for each value gives the following:
FL 270 = 010° at 5 kt. and –25°C
Compare the temperature to standard. Compute the standard temperature by subtracting the lapse rate at FL 270 [(27,000 ÷ 1,000) × 2° = 54°] from sea level standard of 15°C, which is –39°C. The interpolated temperature for FL 270 is approximately 14°C (39 – 25) warmer than standard. Due to variations, temperature is ISA +13°C. (ISA is the International Standard Atmosphere. ISA temperature at sea level is 15°C and decreases at a rate of 2°/1,000 ft. up to 36,000 ft. MSL.)
How well did you know this?
1
Not at all
2
3
4
5
Perfectly
94
Q

Why are downdrafts in a mature thunderstorm hazardous?
A. Downdrafts become warmer than the surrounding air and reverse into an updraft before reaching the surface.
B. Downdrafts converge toward a central location under the storm after striking the surface.
C. Downdrafts are kept cool by cold rain which tends to accelerate the downward velocity.

A

C. Downdrafts are kept cool by cold rain which tends to accelerate the downward velocity.
Answer (C) is correct. (AC 00-6B Chap 19) ( ? )
During the mature stage, cold rain in the downdraft slows compressional heating, and the downdraft remains colder than the surrounding air. Thus, its downward speed is accelerated and may exceed 2,500 fpm.

How well did you know this?
1
Not at all
2
3
4
5
Perfectly
95
Q

Shallow frontal surfaces tend to give
A. extensive cloudiness and large areas of precipitation.
B. heavy precipitation developing along the leading edge of the front.
C. areas of cloudiness and scattered precipitation.

A

A. extensive cloudiness and large areas of precipitation.
Answer (A) is correct. (AC 00-6B Chap 10) ( ? )
Shallow frontal surfaces tend to give extensive cloudiness with large precipitation areas.

How well did you know this?
1
Not at all
2
3
4
5
Perfectly
96
Q

Visibility above a haze layer is poor when looking
A. forward.
B. upward.
C. downward.

A

C. downward.
Answer (C) is correct. (AC 00-6B Chap 16) ( ? )
Haze occurs in stable air and is usually only a few thousand feet thick, but sometimes may extend as high as 15,000 feet. Downward visibility above a haze layer is poor.

How well did you know this?
1
Not at all
2
3
4
5
Perfectly
97
Q

Stratus clouds are known for
A. little or no turbulence with hazardous icing at temperatures near or below freezing.
B. some turbulence and not significant icing.
C. some turbulence and small amounts of icing.

A

A. little or no turbulence with hazardous icing at temperatures near or below freezing.
Answer (A) is correct. (AC 00-6B Chap 3) ( ? )
Stratus clouds are known for very little turbulence but can pose a serious icing problem if temperatures are near or below freezing.

How well did you know this?
1
Not at all
2
3
4
5
Perfectly
98
Q

Which type clouds are indicative of very strong turbulence?
A. Standing lenticular.
B. Cirrocumulus.
C. Nimbostratus.

A

A. Standing lenticular.
Answer (A) is correct. (AC 00-57) ( ? )
Standing lenticular clouds are formed on the crests of mountain waves. Wind can be quite strong blowing through these clouds, and their presence is a good indication of very strong turbulence.

How well did you know this?
1
Not at all
2
3
4
5
Perfectly
99
Q

Any wind blowing down an incline where the incline is influential in causing the wind is a
A. katabatic wind.
B. land breeze.
C. valley wind.

A

A. katabatic wind.
Answer (A) is correct. (AC 00-6A Chap 4) ( ? )
A katabatic wind is any wind blowing down an incline when the incline is influential in causing the wind. Mountain wind is a katabatic wind.

How well did you know this?
1
Not at all
2
3
4
5
Perfectly
100
Q

What condition is reported at Childress (KCDS)?
A. Heavy rain showers began 42 minutes after the hour.
B. The ceiling is solid overcast at an estimated 1,800 feet above sea level.
C. Light rain showers.
See figure 145

A

C. Light rain showers.
Answer (C) is correct. (AC 00-45H Chap 3) ( ? )
In the METAR for KCDS is the code –SHRA, which means there are light (–) showers (SH) and the precipitation type is rain (RA).

How well did you know this?
1
Not at all
2
3
4
5
Perfectly
101
Q

The Hazardous Inflight Weather Advisory Service (HIWAS) is a continuous broadcast over selected VORs of
A. SIGMETs, CONVECTIVE SIGMETs, AIRMETs, Wind Shear Advisories, and Severe Weather Forecast Alerts (AWW).
B. Wind Shear Advisories, Radar Weather Reports, SIGMETs, CONVECTIVE SIGMETs, AIRMETs, and Center Weather Advisories (CWA).
C. SIGMETs, CONVECTIVE SIGMETs, AIRMETs, Severe Weather Forecast Alerts (AWW), and Center Weather Advisories (CWA).

A

C. SIGMETs, CONVECTIVE SIGMETs, AIRMETs, Severe Weather Forecast Alerts (AWW), and Center Weather Advisories (CWA).
Answer (C) is correct. (AC 00-45H Chap 1) ( ? )
The Hazardous Inflight Weather Advisory Service (HIWAS) is a continuous broadcast service over selected VORs of in-flight weather advisories, i.e., SIGMETs, convective SIGMETs, AIRMETs, severe weather forecast alerts (AWW), and center weather advisories (CWA).

How well did you know this?
1
Not at all
2
3
4
5
Perfectly
102
Q

Hazardous vortex turbulence that might be encountered behind large aircraft is created only when that aircraft is
A. using high power settings.
B. operating at high airspeeds.
C. developing lift.

A

C. developing lift.
Answer (C) is correct. (AIM Para 7-3-1) ( ? )
Lift is generated by the creation of a pressure differential over the wing’s, or other airfoil’s, surface. The lowest pressure occurs over the upper wing surface and the highest pressure under the wing. This pressure differential triggers the roll up of the airflow aft of the wing resulting in swirling air masses trailing downstream from the wingtips. An airplane’s wake consists of two counter-rotating vortices.

How well did you know this?
1
Not at all
2
3
4
5
Perfectly
103
Q

Isobars on a surface weather chart represent lines of equal pressure
A. at a given atmospheric pressure altitude.
B. at the surface.
C. reduced to sea level.

A

C. reduced to sea level.
Answer (C) is correct. (AC 00-45H Chap 5) ( ? )
Isobars on a surface analysis (weather) chart represent the sea level pressure pattern, usually spaced at 4-mb, or 4-hPa, intervals. This allows a comparison to be made of pressure reports by stations at various altitudes.

How well did you know this?
1
Not at all
2
3
4
5
Perfectly
104
Q

Temperature and radiation variations over land with a clear sky typically lead to
A. temperature reaching a maximum closer to noon than to sunset.
B. outgoing terrestrial radiation peaking at noon.
C. minimum temperature occurring after sunrise.

A

C. minimum temperature occurring after sunrise.
Answer (C) is correct. (AC 00-6B) ( ? )
At night, heating is absent, but terrestrial radiation continues cooling the earth’s surface. Cooling continues until shortly after sunrise, when incoming solar radiation once again exceeds outgoing terrestrial radiation. Minimum surface air temperature usually occurs shortly after sunrise.

How well did you know this?
1
Not at all
2
3
4
5
Perfectly
105
Q

Which conditions are necessary for the formation of upslope fog?
A. Rain falling through stratus clouds and a 10- to 25-knot wind moving the precipitation up the slope.
B. Moist, stable air being moved over gradually rising ground by a wind.
C. A clear sky, little or no wind, and 100 percent relative humidity.

A

B. Moist, stable air being moved over gradually rising ground by a wind.
Answer (B) is correct. (AC 00-6B Chap 16) ( ? )
Upslope fog is formed by moist, stable air being cooled as it is gradually moved up sloping terrain by a wind. Once the upslope wind ceases, the fog dissipates.

How well did you know this?
1
Not at all
2
3
4
5
Perfectly
106
Q

When will frost most likely form on aircraft surfaces?
A. On clear nights with stable air and light winds.
B. On overcast nights with freezing drizzle precipitation.
C. On clear nights with convective action and a small temperature/dewpoint spread.

A

A. On clear nights with stable air and light winds.
Answer (A) is correct. (FAA-H-8083-25B Chap 12) ( ? )
Frost will most likely form on aircraft surfaces on clear nights with stable air and light winds. These are conditions that in all other respects make weather ideal for flying.

How well did you know this?
1
Not at all
2
3
4
5
Perfectly
107
Q

Constant Pressure Analysis Charts contain contours, isotherms, and some contain isotachs. The contours depict
A. highs, lows, troughs, and ridges on the surface.
B. ridges, lows, troughs, and highs aloft.
C. highs, lows, troughs, and ridges corrected to MSL.

A

B. ridges, lows, troughs, and highs aloft.
Answer (B) is correct. (AC 00-45H Chap 4) ( ? )
Constant pressure analysis charts contain contours and isotherms, and some contain isotachs. Contours are lines of equal height and depict highs, lows, troughs, and ridges aloft in the same manner as isobars on the surface chart.

How well did you know this?
1
Not at all
2
3
4
5
Perfectly
108
Q
On final approach to the airport, airplane in position #5 would experience
A.	decreased ground speed.
B.	downdraft.
C.	poor performance.
See Figure 144
A

C. poor performance.
Answer (C) is correct. (AC 00-54) ( ? )
Groundspeed will increase while indicated airspeed and lift will decrease, potentially leading to a stall with loss of altitude and landing short of the runway.

How well did you know this?
1
Not at all
2
3
4
5
Perfectly
109
Q

If you take off behind a heavy jet that has just landed, you should plan to lift off
A. prior to the point where the jet touched down.
B. beyond the point where the jet touched down.
C. at the point where the jet touched down and on the upwind edge of the runway.

A

B. beyond the point where the jet touched down.
Answer (B) is correct. (AIM Para 7-3-6) ( ? )
You should ensure that an interval of at least 2 min. has elapsed before your takeoff behind a heavy jet that has just landed. Note the point where the heavy jet touched down and plan to lift off beyond that point. The vortices developed by the heavy jet will no longer be generated when lift is no longer produced (which is at the point of touchdown of the jet’s nose gear).

How well did you know this?
1
Not at all
2
3
4
5
Perfectly
110
Q

Which type storms are most likely to produce funnel clouds or tornadoes?
A. Cold front or squall line thunderstorms.
B. Storms associated with icing and supercooled water.
C. Air mass thunderstorms.

A

A. Cold front or squall line thunderstorms.
Answer (A) is correct. (FAA-H-8083-25B Chap 12) ( ? )
Funnel clouds or tornadoes normally form with steady-state thunderstorms associated with cold fronts or squall lines. The mature stage updrafts in a steady-state thunderstorm are stronger and last longer than those in air mass storms and may persist for several hours.

How well did you know this?
1
Not at all
2
3
4
5
Perfectly
111
Q

Maximum Turbulence Potential forecasts
A. exhibit the same accuracy during all 24 hours of the day.
B. are only as accurate as the computer model.
C. display all turbulence well.

A

B. are only as accurate as the computer model.
Answer (B) is correct. (AC 00-45H Chap 5) ( ? )
This product is based on an ensemble of turbulence indicators and, therefore, can capture more diverse sources of turbulence and provide a more reliable forecast than can be provided by a single indicator. The accuracy of the product depends on the accuracy of the computer model output used to create it.

How well did you know this?
1
Not at all
2
3
4
5
Perfectly
112
Q

Moderate CAT is considered likely when the vertical wind shear is
A. 5 knots per 1,000 feet or greater and/or the horizontal wind shear is 40 knots per 150 miles or greater.
B. 10 knots per 1,000 feet or greater and/or the horizontal wind shear is 50 knots per 150 miles or greater.
C. 15 knots per 2,000 feet or greater and/or the horizontal wind shear is 40 knots per 150 miles or greater.

A

A. 5 knots per 1,000 feet or greater and/or the horizontal wind shear is 40 knots per 150 miles or greater.
Answer (A) is correct. (AC 00-30C) ( ? )
Wind shear occurs in all directions, but, for convenience, it is measured along vertical and horizontal axes, thus becoming horizontal and vertical wind shear. Moderate clear air turbulence (CAT) is considered likely when the vertical wind shear is 5 knots per 1,000 feet or greater and/or the horizontal wind shear is 40 knots per 150 miles or greater.

How well did you know this?
1
Not at all
2
3
4
5
Perfectly
113
Q

Which is one of the three key elements in an effective clear air turbulence (CAT) Avoidance System?
A. A dedicated planning/dispatch function.
B. An ATC-based communications system.
C. Appropriate training every 6 months.

A

A. A dedicated planning/dispatch function.
Answer (A) is correct. (AC 00-30C) ( ? )
There are three key elements in an effective CAT avoidance system: (1) an appropriate initial and recurrent training program, (2) a dedicated planning/dispatch function, and (3) a fully supported operational implementation of a PIREP/communications system (not ATC-based).

How well did you know this?
1
Not at all
2
3
4
5
Perfectly
114
Q

If a SIGMET alert is announced, how can information contained in the SIGMET be obtained?
A. By contacting a weather watch station.
B. By contacting the nearest FSS.
C. ATC will announce the hazard and advise when information will be provided in the FSS broadcast.

A

B. By contacting the nearest FSS.
Answer (B) is correct. (AIM Para 7-1-11) ( ? )
Pilots, upon hearing the alert notice, if they have not received the advisory or are in doubt, should contact the nearest FSS and ascertain whether the advisory is pertinent to their flights.

How well did you know this?
1
Not at all
2
3
4
5
Perfectly
115
Q

The minimums for the nonprecision approach at KAMA are 3/4 mile visibility and 400 feet. When operating under Part 121, can the pilot legally execute the approach with the given METAR data?
A. No, they do not meet the minimum visibility requirements.
B. Yes, they meet the minimum visibility requirements.
C. No, they do not meet the minimum ceiling requirements.
See figure 145

A

B. Yes, they meet the minimum visibility requirements.
Answer (B) is correct. (AC 00-45H Chap 3) ( ? )
The visibility given in the METAR is 3/4 mile, equal to the minimum for the approach. Part 121 allows the approach to continue past the final approach fix if a weather report indicates the visibility is equal to or greater than the approach minimum, irrespective of the ceiling level.

How well did you know this?
1
Not at all
2
3
4
5
Perfectly
116
Q

What action is recommended when encountering turbulence due to a wind shift associated with a sharp pressure trough?
A. Establish a straight course across the storm area.
B. Climb or descend to a smoother level.
C. Increase speed to get out of the trough as soon as possible.

A

A. Establish a straight course across the storm area.
Answer (A) is correct. (AC 00-30B) ( ? )
If turbulence is encountered in an abrupt wind shift associated with a sharp pressure trough line, you should establish a straight course across the storm area. This action is recommended because the turbulence normally is parallel to the trough and by crossing the storm area you will minimize the turbulence.

117
Q

Freezing drizzle often forms by the
A. collision-coalescence process.
B. condensation-collision process.
C. condensation-condensation process.

A

A. collision-coalescence process.
Answer (A) is correct. (AC 00-6B Chap 14) ( ? )
Freezing drizzle is more commonly formed by a process known as the collision-coalescence process. Some droplets in a cloud grow to approximately 30 micrometers in diameter. Then they begin to settle and fall fast enough so that they collide with some smaller droplets. If the droplets coalesce, the result is a larger droplet, which now has an even better chance of capturing smaller droplets. This process can produce drizzle-size drops in a supercooled cloud, usually near the top, where the larger droplets generally are found in any cloud.

118
Q

Airborne weather radar is installed to help the crew
A. avoid storm turbulence and hail.
B. penetrate weather between storm cells.
C. avoid severe weather.

A

C. avoid severe weather.
Answer (C) is correct. (FAA-H-8083-25B Chap 13) ( ? )
Airborne radar is equipment carried by aircraft to locate and avoid severe weather.

119
Q

What type turbulence should be reported when it causes slight, rapid, and somewhat rhythmic bumpiness without appreciable changes in attitude or altitude, less than one-third of the time?
A. Moderate chop.
B. Moderate turbulence.
C. Occasional light chop.

A

C. Occasional light chop.
Answer (C) is correct. (AIM Para 7-1-25) ( ? )
Light chop is turbulence that causes slight, rapid, and somewhat rhythmic bumpiness without appreciable changes in attitude or altitude. Occasional is defined as occurring less than one-third of the time.

120
Q
What will be the wind and temperature trend for an STL-MEM-MSY flight at FL 330?
A.	Wind shift from west to north.
B.	Temperature increase 5°C.
C.	Windspeed decrease.
See figure 149
A

C. Windspeed decrease.
Answer (C) is correct. (AC 00-45H Chap 7) ( ? )
To determine the wind and temperature trend, the winds and temperatures aloft must be interpolated for FL 330. The table below shows data for FL 300 and FL 340 and the interpolated data for FL 330.

FL 300
FL 340
FL 330
STL
265435
265744
265642
MEM
261934
262144
262142
MSY
990034
990043
990041
During this flight, the wind speed decreases from 56 kt. to light and variable.
121
Q

What minimum thickness of cloud layer is indicated if precipitation is reported as light or greater intensity?
A. A thickness which allows the cloud tops to be higher than the freezing level.
B. 2,000 feet thick.
C. 4,000 feet thick.

A

C. 4,000 feet thick.
Answer (C) is correct. (AC 00-6B Chap 14) ( ? )
When arriving at or departing from an airport reporting precipitation of light or greater intensity, expect clouds to be at least 4,000 ft. thick.

122
Q

An aircraft that encounters a headwind of 40 knots, within a microburst, may expect a total shear across the microburst of
A. 40 knots.
B. 80 knots.
C. 90 knots.

A

B. 80 knots.
Answer (B) is correct. (AIM Para 7-1-26) ( ? )
If a headwind in a microburst is 40 kt., the wind will be going in the opposite direction on the other side of a microburst at presumably the same 40 kt., resulting in an 80-kt. shear (headwind to tailwind change for a traversing aircraft) across the microburst.

123
Q

The Federal Aviation Administration’s Flight Information Service Data Link (FISDL) products, such as ground radar precipitation maps,
A. are not appropriate for finding a path through a weather hazard area.
B. may be used to find a path through a weather hazard area.
C. may be used instead of the aircraft radar.

A

A. are not appropriate for finding a path through a weather hazard area.
Answer (A) is correct. (AC 00-63A) ( ? )
FISDL products, such as ground-based radar precipitation maps, are not appropriate for use in tactical severe weather avoidance, such as negotiating a path through a weather hazard area. FISDL supports strategic weather decision making, such as route selection to avoid a weather hazard area in its entirety.

124
Q

To avoid the wingtip vortices of a departing jet airplane during takeoff, the pilot should
A. climb above and stay upwind of the jet airplane’s flight path.
B. remain below the flight path of the jet airplane.
C. lift off at a point well past the jet airplane’s flight path.

A

A. climb above and stay upwind of the jet airplane’s flight path.
Answer (A) is correct. (AIM Para 7-3-6) ( ? )
A departing jet airplane’s rotation point is where full vortices are first generated. On your takeoff, rotate prior to that point. Climb above and remain upwind of the jet airplane’s flight path to avoid the descending wingtip vortices. Avoid subsequent headings which will cross below and behind the jet airplane (i.e., the jet will climb faster than a propeller airplane).

125
Q

Land surface heating on the coast results in
A. a land breeze.
B. a sea breeze.
C. a chinook wind.

A

B. a sea breeze.
Answer (B) is correct. (AC 00-6B Chap 9) ( ? )
During the day, the wind that blows from the cool water to warm the land is a sea breeze.

126
Q

KFTW UA/OV DFW/TM 1645/FL100/TP PA30/SK SCT031- TOP043/BKN060-TOP085/OVC097- TOPUNKN/WX FV00SM RA/TA 07.
This pilot report to Fort Worth (KFTW) indicates

A. the aircraft is in light rain.
B. the ceiling at KDFW is 6,000 feet.
C. that the top of the ceiling is 4,300 feet.

A

B. the ceiling at KDFW is 6,000 feet.
Answer (B) is correct. (AC 00-45H Chap 3) ( ? )
A ceiling is defined as the height (AGL) of the lowest broken or overcast layer aloft or vertical visibility into an obscuration. The PIREP reports the lowest broken layer at 6,000 ft. MSL (BKN060). This indicates the ceiling is 6,000 ft. A ceiling is normally defined as the height above ground, but in a PIREP all altitudes are MSL unless otherwise indicated.

127
Q

On initial climbout after takeoff with the autopilot engaged, you encounter icing conditions. In this situation you can expect
A. the autopilot to hold the vertical speed, only if the anti-icing boots are working.
B. increased airflow under the wings to prevent the accumulation of ice.
C. ice to accumulate on the underside of the wings due to the higher AOA.

A

C. ice to accumulate on the underside of the wings due to the higher AOA.
Answer (C) is correct. (AC 91-74B) ( ? )
Airplanes are vulnerable to ice accumulation during the initial climbout in icing conditions because lower speeds often translate into a higher Angle of Attack (AOA). This exposes the underside of the airplane and its wings to the icing conditions and allows ice to accumulate further aft than it would in cruise flight.

128
Q

Clear ice generally forms in outside temperature ranges of
A. 0 to -10°C.
B. colder than -25°C.
C. -15° to -25°C.

A

A. 0 to -10°C.
Answer (A) is correct. (AC 00-6B Chap 18) ( ? )
Clear ice generally forms when outside temperatures range from 0°C to -10°C

129
Q

Which condition is present when a local parcel of air is stable?
A. The parcel of air cannot be forced uphill.
B. The parcel of air resists convection.
C. As the parcel of air moves upward, its temperature becomes warmer than the surrounding air.

A

B. The parcel of air resists convection.
Answer (B) is correct. (AC 00-6B Chap 12) ( ? )
Stable air resists convection. If a parcel of air is lifted, it is cooler than the surrounding air and will return to its original position when the lifting force is removed. Spontaneous convection is impossible in stable air.

130
Q

A tropical cyclone is
A. a hurricane force storm in the Pacific Ocean southwest of Hawaii with highest sustained winds of 65 knots or higher.
B. a tropical storm in the north Atlantic west of Newfoundland with highest sustained winds of 25 through 33 knots.
C. a tropical depression in the Northern Pacific near Dutch Harbor with sustained winds of 63 knots.

A

A. a hurricane force storm in the Pacific Ocean southwest of Hawaii with highest sustained winds of 65 knots or higher.
Answer (A) is correct. (AC 00-6B Chap 2) ( ? )
A hurricane force storm in the area stated, with sustained winds of 64 knots or higher, may be classified as a tropical cyclone.
Tropical cyclone is a general term for a rotating, organized system of clouds and thunderstorms that originates over tropical or subtropical waters. The tropical cyclone international classifications include:

1.
Tropical Depression – highest sustained winds up to 33 knots
2.
Tropical Storm – highest sustained winds of 34 to 63 knots
3.
Hurricane, Cyclone, or Typhoon – highest sustained winds 64 knots or more

131
Q
What approximate wind direction, speed, and temperature (relative to ISA) are expected for a flight over MKC at FL 260?
A.	260° magnetic; 42 knots; ISA +9°C.
B.	260° true; 43 knots; ISA +10°C.
C.	260° true; 45 knots; ISA –10°C.
See figure 149
A
B.	260° true; 43 knots; ISA +10°C.
Answer (B) is correct. (AC 00-45H Chap 7) ( ? )
For conditions at FL 260 over MKC in Fig. 149, interpolate between values at FL 240 and FL 300. First decode the two given flight levels:
FL 240
=
260°
at
38 kt.
and
–21°C
FL 300
=
260°
at
50 kt.
and
–36°C
Difference
=
0°
at
12 kt.
and
–15°C
Interpolation for each value gives the following:
FL 260 = 260° at 42 kt. and –26°C
To compare the temperature to standard, subtract the lapse rate at FL 260 [(26,000 ÷ 1,000) × 2° = 52°] from sea level standard of 15°C to get FL 260 standard of –37°C. The interpolated temperature for FL 260 of about –26°C is 11°C warmer than standard. The best answer is 260° true; 43 kt.; ISA +10. (ISA is the International Standard Atmosphere. ISA temperature at sea level is 15°C and decreases at a rate of 2°/1,000 ft. up to 36,000 ft. MSL.)
132
Q

Which feature is associated with the tropopause?
A. Absolute upper limit of cloud formation.
B. Abrupt change of temperature lapse rate.
C. Absence of wind and turbulence.

A

B. Abrupt change of temperature lapse rate.
Answer (B) is correct. (AC 00-6B Chap 1) ( ? )
The tropopause is characterized by an abrupt change in temperature lapse rate. The temperature above the tropical tropopause increases with height, and the temperature above the polar tropopause remains about constant with height.

133
Q

The air is 100% saturated with vapor at
A. the dew-point temperature.
B. the humidity index.
C. wet-bulb temperature.

A

A. the dew-point temperature.
Answer (A) is correct. (AC 00-6B Chap 3) ( ? )
Relative humidity is expressed as a percent. It relates the actual water vapor present to that which could be present. Air with 100% humidity is saturated and is the dew-point temperature.

134
Q

Large areas of land
A. minimize temperature variations.
B. tend to increase temperature variations.
C. do not influence the troposphere.

A

B. tend to increase temperature variations.
Answer (B) is correct. (AC 00-6B Chap 2) ( ? )
Large areas of land increase temperature variations. While a daily maximum and minimum temperature change may be only 10°C over water, a change of up to 50°C can occur over land.

135
Q

How will the aircraft in position 4 be affected by a microburst encounter?
A. Performance decreasing with a tailwind and downdraft.
B. Performance decreasing with a headwind and downdraft.
C. Performance increasing with a tailwind and updraft.
See figure 144

A

A. Performance decreasing with a tailwind and downdraft.
Answer (A) is correct. (AC 00-54) ( ? )
The aircraft at position 4 on Fig. 144 is affected by the microburst’s causing a decrease in airplane performance due to a tailwind and downdraft.

136
Q

In comparison to an approach in a moderate headwind, which is an indication of a possible wind shear due to a decreasing headwind when descending on the glide slope?
A. Higher pitch attitude is required.
B. Less power is required.
C. Lower descent rate is required.

A

A. Higher pitch attitude is required.
Answer (A) is correct. (AC 00-54) ( ? )
A decreasing headwind shear will decrease indicated airspeed and performance capability. Due to a loss of airspeed, the airplane will tend to pitch down. Thus, a higher pitch attitude is required.

137
Q

A squall is a sudden increase of at least 16 knots in average wind speed to a sustained speed of
A. 26 knots or more for at least 1 minute.
B. 22 knots or more for at least 1 minute.
C. 20 knots or more for at least 1 minute.

A

B. 22 knots or more for at least 1 minute.
Answer (B) is correct. (NWS Glossary)
A squall is a sudden increase of at least 16 kt. in average wind speed to a sustained speed of 22 kt. or more for at least 1 minute. Squalls are reported in the METAR as weather phenomena by the code SQ.

138
Q

A strong wind shear can be expected
A. where the horizontal wind shear is 15 knots, in a distance equal to 2.5° longitude.
B. on the low pressure side of a 100-knot jetstream core.
C. if the 5°C isotherms are spaced 100 NM or closer together.

A

B. on the low pressure side of a 100-knot jetstream core.
Answer (B) is correct. (AC 00-30C) ( ? )
Wind speed decreases outward from the jet stream core, and the rate of decrease is greater on the polar (low-pressure) side than on the equatorial (high-pressure) side. Thus, a strong wind shear can be expected on the low-pressure side of a 100-kt. jet stream core.

139
Q

Which weather condition is present when a tropical storm is upgraded to a typhoon?
A. Barometric pressure at the center is 35 mb lower than the surrounding pressure.
B. The storm has sustained winds of 64 knots or more.
C. Winds around the pressure center have averaged 48 knots or more for 48 continuous hours.

A

B. The storm has sustained winds of 64 knots or more.
Answer (B) is correct. (AC 00-6B Chap 21) ( ? )
To be categorized as a hurricane or a typhoon, the storm must have highest sustained winds of 64 knots or more.

140
Q

Which type wind flows downslope becoming warmer and dryer?
A. Katabatic wind.
B. Valley wind.
C. Land breeze.

A

A. Katabatic wind.
Answer (A) is correct. (AC 00-6A Chap 4) ( ? )
Katabatic wind forms because cold, heavy air spills down sloping terrain displacing warmer, less dense air ahead of it. The cold, heavy air is adiabatically heated and dried as it flows downslope because of heating due to the downward air being compressed by increasing pressure.

141
Q

What is the recommended technique to counter the loss of airspeed and resultant lift from wind shear?
A. Avoid overstressing the aircraft, “pitch to airspeed,” and apply maximum power.
B. Lower the pitch attitude and regain lost airspeed.
C. Maintain, or increase, pitch attitude and accept the lower-than-normal airspeed indications.

A

C. Maintain, or increase, pitch attitude and accept the lower-than-normal airspeed indications.
Answer (C) is correct. (AC 00-54) ( ? )
A loss of airspeed and resultant lift indicates an increasing tailwind (or decreasing headwind) shear. Successful recovery from a wind-shear encounter requires maintaining or increasing pitch attitude and accepting lower-than-normal airspeed indications.

142
Q

Which action is recommended if jet stream turbulence is encountered with a direct headwind or tailwind?
A. Change course to fly on the polar side of the jet stream.
B. Change altitude or course to avoid a possible elongated turbulent area.
C. Increase airspeed to get out of the area quickly.

A

B. Change altitude or course to avoid a possible elongated turbulent area.
Answer (B) is correct. (AC 00-30B) ( ? )
If jet stream turbulence with direct tailwinds or headwinds is encountered, a change of altitude or course should be initiated since these turbulent areas are elongated with the wind and are shallow and narrow.

143
Q

Which system in the Convective SIGMET listing has the potential of producing the most severe storm?
A. The storms in Texas and Oklahoma.
B. The isolated storm 50 miles northeast of Memphis (MEM).
C. The storms in Colorado, Kansas, and Oklahoma.
See figure 148

A

B. The isolated storm 50 miles northeast of Memphis (MEM).
Answer (B) is correct. (AC 00-45H Chap 3) ( ? )
The isolated thunderstorm 50 mi. northeast of MEM is a VIP level 5 (intense) storm. With a top above 45,000 ft., extensive vertical development is taking place.

144
Q

In which meteorological conditions can frontal waves and low pressure areas form?
A. Warm fronts or occluded fronts.
B. Cold front occlusions.
C. Slow-moving cold fronts or stationary fronts.

A

C. Slow-moving cold fronts or stationary fronts.
Answer (C) is correct. (AC 00-6B Chap 10) ( ? )
Frontal waves and areas of low pressure usually form on slow-moving cold fronts or on stationary fronts.

145
Q

Which arctic flying hazard is caused when a cloud layer of uniform thickness overlies a snow or ice covered surface?
A. Blowing snow.
B. Ice fog.
C. Whiteout.

A

C. Whiteout.
Answer (C) is correct. (AC 00-6B Chap 22) ( ? )
Whiteout is a visibility-restricting phenomenon that occurs in the Arctic when a cloud layer of uniform thickness overlies a snow- or ice-covered surface. Parallel rays of the sun are broken up and diffused when passing through the cloud layer so that they strike the snow surface from many angles, resulting in a loss of depth perception.

146
Q

Which INITIAL flight deck indications should a pilot be aware of when a constant tailwind shears to a calm wind?
A. Altitude increases; pitch and indicated airspeed decrease.
B. Altitude, pitch, and indicated airspeed increase.
C. Altitude, pitch, and indicated airspeed decrease.

A

B. Altitude, pitch, and indicated airspeed increase.
Answer (B) is correct. (AC 00-54) ( ? )
When a constant tailwind shears to a calm wind, the shear increases indicated airspeed and thus increases performance. The airplane tends to pitch up and gain altitude.

147
Q

Which condition produces weather on the lee side of a large lake?
A. Warm air flowing over a cool lake may produce rain showers.
B. Cold air flowing over a warmer lake may produce advection fog.
C. Warm air flowing over a colder lake may produce fog.

A

C. Warm air flowing over a colder lake may produce fog.
Answer (C) is correct. (AC 00-6B Chap 16) ( ? )
Warm air flowing over a colder lake may become saturated by evaporation from the water while also becoming cooler in the low levels by contact with the cooler water. Fog often becomes extensive and dense to the lee side of the lake. The lee side is the side to which the wind is going.

148
Q

If squalls are reported at the destination airport, what wind conditions exist?
A. Rapid variation in wind direction of at least 20° and changes in speed of at least 10 knots between peaks and lulls.
B. Sudden increases in wind speed of at least 15 knots to a sustained wind speed of 20 knots, lasting for at least 1 minute.
C. A sudden increase in wind speed of at least 16 knots, the speed rising to 22 knots or more for 1 minute or longer.

A

C. A sudden increase in wind speed of at least 16 knots, the speed rising to 22 knots or more for 1 minute or longer.
Answer (C) is correct. (NWS Glossary)
A squall is a sudden increase in wind speed of at least 16 kt., the speed rising to 22 kt. or more, and lasting for at least 1 min.

149
Q
What is the forecasted wind direction, speed, and temperature over ABI at 30,000 feet?
A.	240°, 8 knots, –33 °C.
B.	240°, 8 knots, 33 °C.
C.	240°, 108 knots, –33 °C.
See figure 149
A

A. 240°, 8 knots, –33 °C.
Answer (A) is correct. (AIM Para 7-1-8) ( ? )
Find the winds aloft forecast for ABI on the top line of Fig. 149. Move to the right until you reach the column marked 30000, where it reads 240833. The 24 indicates a wind direction of 240°, 08 indicates a wind speed of 8 kt., and since all temperatures are negative above FL 240, the 33 indicates a –33°C temperature.

150
Q

What feature is normally associated with the cumulus stage of a thunderstorm?
A. Beginning of rain at the surface.
B. Frequent lightning.
C. Continuous updraft.

A

C. Continuous updraft.
Answer (C) is correct. (AC 00-6B Chap 19) ( ? )
During the cumulus stage of a thunderstorm, the cumulus cloud is building, and there are continuous updrafts.

151
Q

Convective SIGMETs are issued for severe thunderstorms
A. occurring for more than one hour of the valid period.
B. occurring for more than 30 minutes of the valid period.
C. affecting more than 1,000 square miles of airspace.

A

B. occurring for more than 30 minutes of the valid period.
Answer (B) is correct. (AC 00-45H) ( ? )
Convective SIGMETs are issued for embedded or severe thunderstorms expected to occur for more than 30 minutes during the valid period regardless of the size of the area.

152
Q

At which location does Coriolis force have the least effect on wind direction?
A. At the Equator.
B. Middle latitudes (30° to 60°).
C. At the poles.

A

A. At the Equator.
Answer (A) is correct. (AC 00-6B Chap 7) ( ? )
The Coriolis force affects the wind direction everywhere except immediately at the Equator.

153
Q

Thrust is being managed to maintain desired indicated airspeed and the glide slope is being flown. Which characteristics should be observed when a tailwind shears to a constant headwind?
A. PITCH ATTITUDE: Increases. VERTICAL SPEED: Increases. INDICATED AIRSPEED: Decreases, then increases to approach speed.
B. PITCH ATTITUDE: Decreases. VERTICAL SPEED: Decreases. INDICATED AIRSPEED: Decreases, then increases to approach speed.
C. PITCH ATTITUDE: Increases. VERTICAL SPEED: Decreases. INDICATED AIRSPEED: Increases, then decreases.

A

C. PITCH ATTITUDE: Increases. VERTICAL SPEED: Decreases. INDICATED AIRSPEED: Increases, then decreases.
Answer (C) is correct. (AC 00-54) ( ? )
When a tailwind shears to a constant headwind, the indicated airspeed will initially increase, then decrease. The vertical speed will decrease, and pitch attitude will increase. Initially, thrust will be reduced to slow the airspeed, then increased as more power and a slower rate of descent are needed to maintain the glide slope in a constant headwind.

154
Q

METAR KSPS 131757Z 09014KT 6SM -RA SCT025 OVC090 24/22 A3005.
SPECI KSPS 131820Z 01025KT 3SM +RA FC OVC015 22/21 A3000.

What change took place at Wichita Falls (KSPS) between 1757 and 1820 UTC?

A. The rain became lighter.
B. Atmospheric pressure increased.
C. A funnel cloud was observed.

A

C. A funnel cloud was observed.
Answer (C) is correct. (AC 00-45H Chap 3) ( ? )
The special report (SPECI) for KSPS at 1820 UTC reports that a funnel cloud (FC) was observed.

155
Q

Clear air turbulence (CAT) associated with a mountain wave may extend as far as
A. 1,000 miles or more downstream of the mountain.
B. 5,000 feet above the tropopause.
C. 100 miles or more upwind of the mountain.

A

B. 5,000 feet above the tropopause.
Answer (B) is correct. (AC 00-6A Chap 13) ( ? )
Mountain waves can create clear air turbulence (CAT). Mountain wave CAT may extend from the mountain crests to as high as 5,000 ft. above the tropopause and can range 100 mi. or more downstream from the mountains.

156
Q

If the ambient temperature is warmer than standard at FL 350, what is the density altitude compared to pressure altitude?
A. Impossible to determine without information on possible inversion layers at lower altitudes.
B. Lower than pressure altitude.
C. Higher than pressure altitude.

A

C. Higher than pressure altitude.
Answer (C) is correct. (AC 00-6B Chap 5) ( ? )
Density altitude, by definition, is pressure altitude adjusted for nonstandard temperature. When the ambient air temperature is warmer than standard, the expanded air is lighter in weight per unit volume than on a cold day, and the pressure levels are raised. Thus, the pressure level where the altimeter indicates FL 350 is higher on a warm day than under standard conditions.

157
Q

Frozen dew is
A. white and opaque.
B. hard and transparent.
C. hard and opaque.

A

B. hard and transparent.
Answer (B) is correct. (AC 00-6A Chap 5) ( ? )
Sometimes dew forms and later freezes. Frozen dew is easily distinguished from frost. Frozen dew is hard and transparent, while frost is white and opaque.

158
Q

What is the single source reference that contains information regarding volcanic eruption, turbulence, and icing conditions for a specific region?
A. In-Flight Weather Advisories.
B. Low-Level Significant Weather Prog Chart.
C. Ceiling and Visibility Analysis.

A

A. In-Flight Weather Advisories.
Answer (A) is correct. (AC 00-45H Chap 3) ( ? )
In-flight aviation weather advisories provide information on volcanic eruption, turbulence, and icing conditions for a specific region.

159
Q

Summer thunderstorms in the arctic region will generally move
A. southwest to northeast with the jetstream flow.
B. directly north to south with the low-level polar airflow.
C. northeast to southwest in polar easterlies.

A

C. northeast to southwest in polar easterlies.
Answer (C) is correct. (AC 00-6B Chap 22) ( ? )
During summer afternoons, scattered cumulus clouds forming over the interior of the arctic region occasionally grow into thunderstorms. These thunderstorms generally move from northeast to southwest in the polar easterlies, which is opposite the general movement in midlatitudes.

160
Q

Atmospheric pressure changes due to a thunderstorm will be at the lowest value
A. when the thunderstorm is approaching.
B. during the downdraft and heavy rain showers.
C. immediately after the rain showers have stopped.

A

A. when the thunderstorm is approaching.
Answer (A) is correct. (AC 00-24C) ( ? )
Pressure usually falls rapidly with the approach of a thunderstorm, then rises sharply with the onset of the first gust and arrival of the cold downdraft and heavy rain showers, falling back to normal as the storm moves on.

161
Q

Where are jetstreams normally located?
A. In a break in the tropopause where intensified temperature gradients are located.
B. In areas of strong low pressure systems in the stratosphere.
C. In a single continuous band, encircling the Earth, where there is a break between the equatorial and polar tropopause.

A

A. In a break in the tropopause where intensified temperature gradients are located.
Answer (A) is correct. (AC 00-30C) ( ? )
The jet stream is a narrow, shallow, meandering river of maximum winds extending around the Earth in a wavelike pattern. The jet stream typically occurs in a break in the tropopause in an area of intensified temperature gradients characteristic of the break.

162
Q

What is a difference between an air mass thunderstorm and a steady-state thunderstorm?
A. Air mass thunderstorm downdrafts and precipitation retard and reverse the updrafts.
B. Air mass thunderstorms produce precipitation which falls outside of the updraft.
C. Steady-state thunderstorms are associated with local surface heating.

A

A. Air mass thunderstorm downdrafts and precipitation retard and reverse the updrafts.
Answer (A) is correct. (FAA-H-8083-25B Chap 12) ( ? )
The rain associated with an air mass thunderstorm falls through or immediately beside the updraft. Falling precipitation induces frictional drag, retards the updraft, and reverses it to a downdraft. During a steady-state thunderstorm, precipitation falls outside the updraft, allowing the updraft to continue unabated.

163
Q

What is a feature of air movement in a high pressure area?
A. Descending to the surface and then outward.
B. Ascending from the surface high to lower pressure at higher altitudes.
C. Moving outward from the high at high altitudes and into the high at the surface.

A

A. Descending to the surface and then outward.
Answer (A) is correct. (FAA-H-8083-25B Chap 12) ( ? )
The air movement out of a high-pressure area depletes the quantity of air because the air is heavier than the low-pressure air surrounding it. Therefore, high-pressure areas are areas of descending air toward the surface, then outward from the high.

164
Q

Doppler wind measurements indicate that the windspeed change a pilot may expect when flying through the peak intensity of a microburst is approximately
A. 25 knots.
B. 45 knots.
C. 15 knots.

A

B. 45 knots.
Answer (B) is correct. (AC 00-54) ( ? )
Doppler radar wind measurements indicate that the wind speed change a pilot might expect when flying through the average microburst at its point of peak intensity is about 45 kt. However, microburst wind speed differences of almost 100 kt. have been measured.

165
Q

Adiabatic warming is also described as
A. solar warming.
B. compressional heating.
C. expansional heating.

A

B. compressional heating.
Answer (B) is correct. (AC 00-6B Chap 11) ( ? )
Adiabatic warming (heating) is a process of heating dry air through compression. For example, as air moves down a slope it is compressed, which results in an increase in temperature.

166
Q

Where is the usual location of a thermal low?
A. Over the surface of a dry, sunny region.
B. Over the eye of a hurricane.
C. Over the arctic region.

A

A. Over the surface of a dry, sunny region.
Answer (A) is correct. (AC 00-6B Chap 21) ( ? )
A dry, sunny region becomes quite warm from intense surface heating, thus generating a surface low-pressure area known as a thermal low. Thermal lows are relatively shallow with weak pressure gradients and without well-defined cyclonic circulation.

167
Q

At an indicated altitude of 1,000 feet with a temperature of 32°C, density altitude is
A. 1,400 feet.
B. 3,400 feet.
C. 2,400 feet.

A

B. 3,400 feet.
Answer (B) is correct. (FAA-H-8083-25B Chap 11) ( ? )
Density altitude is calculated using Celsius. In the pressure altitude scale, line up 32°C over 1,000 feet of pressure altitude. The density altitude arrow is pointing to about 3,400 feet.

168
Q
What was the ceiling at Walnut Ridge (KARG)?
A.	2,400 feet AGL.
B.	1,000 feet AGL.
C.	1,000 feet MSL.
See Figure 146
A

B. 1,000 feet AGL.
Answer (B) is correct. (AC 00-45H Chap 3) ( ? )
The ceiling is the lowest broken or overcast layer (measured in hundreds of feet above ground level) or vertical visibility (measured in hundreds of feet) into an obscuration. The ceiling at KARG is reported to be overcast clouds at 1,000 ft. AGL (OVC010).

169
Q

An upper level wind cannot be considered a jet stream unless the wind speed is at least
A. 35 knots.
B. 65 knots.
C. 50 knots.

A

C. 50 knots.
Answer (C) is correct. (AC 00-6A Chap 13) ( ? )
By definition, the concentrated winds must be 50 knots or greater to be classified as a jet stream.

170
Q

Turbulence encountered above 15,000 feet AGL, not associated with cloud formations, should be reported as
A. convective turbulence.
B. high-altitude turbulence.
C. clear air turbulence.

A

C. clear air turbulence.
Answer (C) is correct. (AC 00-6B Chap 17) ( ? )
High-level turbulence encountered above 15,000 ft. AGL, not associated with cumuliform cloudiness, should be reported as clear air turbulence.

171
Q

A hurricane has formed when
A. the highest sustained winds are 64 knots or more.
B. the barometric pressure in the center of the storm is 35 mb lower than pressure outside the weather feature.
C. the winds around a pressure storm have averaged 48 knots for over 48 hours.

A

A. the highest sustained winds are 64 knots or more.
Answer (A) is correct. (AC 00-6B Chap 21) ( ? )
A hurricane or typhoon has the highest sustained winds of 64 knots or more.

172
Q

A PROB40 (PROBability) HHhh group in an International Terminal Aerodrome Forecast (TAF) indicates the probability of
A. thunderstorms or high wind.
B. thunderstorms or other precipitation.
C. precipitation or low visibility.

A

B. thunderstorms or other precipitation.
Answer (B) is correct. (AC 00-45H Chap 7) ( ? )
A PROB40 (PROBability) HHhh group in a TAF indicates the probability of thunderstorms or other precipitation events. The PROB group is used when the occurrence of thunderstorms or precipitation is in the 30% to less than 50% range; thus, the probability value of 40 is appended to the PROB contraction. PROB40 is followed by a four-digit group giving the beginning time (HH) and ending time (hh) of the period during which thunderstorms or precipitation is expected.

173
Q

What weather difference is found on each side of a “dry line”?
A. Stratus versus cumulus clouds.
B. Extreme temperature difference.
C. Dewpoint difference.

A

C. Dewpoint difference.
Answer (C) is correct. (AC 00-6B Chap 10) ( ? )
A “dry line” is common in Western Texas and New Mexico northward over the Plains. Moist air flowing north from the Gulf of Mexico meets the drier and slightly denser air flowing from the southwest. Except for moisture (dew point) differences, there is seldom any significant weather difference found on each side.

174
Q

Where is a common location for an inversion?
A. At the base of cumulus clouds.
B. In the stratosphere.
C. At the tropopause.

A

B. In the stratosphere.
Answer (B) is correct. (AC 00-6B Chap 2) ( ? )
Inversions may occur at any altitude and are common in the stratosphere. An inversion is an increase in temperature with an increase in altitude.

175
Q

When saturated air moves downhill, its temperature increases
A. at a faster rate than dry air because of the release of latent heat.
B. at a slower rate than dry air because condensation releases heat.
C. at a slower rate than dry air because vaporization uses heat.

A

C. at a slower rate than dry air because vaporization uses heat.
Answer (C) is correct. (AC 00-6B Chap 3) ( ? )
When saturated air moves downhill, its temperature increases at a slower rate than dry air because vaporization uses heat known as the “latent heat of vaporization.” The water removal cools the air from which the water is removed.

176
Q

What is a feature of supercooled water?
A. The unstable water drop freezes upon striking an exposed object.
B. The temperature of the water drop remains at 0°C until it impacts a part of the airframe, then clear ice accumulates.
C. The water drop deposits to an ice particle upon impact.

A

A. The unstable water drop freezes upon striking an exposed object.
Answer (A) is correct. (AC 91-74B) ( ? )
Water droplets that are colder than 0°C are considered supercooled. When they strike an exposed object, they freeze on impact. Aircraft icing results if supercooled water strikes an airplane.

177
Q

Cloud bases can usually be estimated by dividing the surface Fahrenheit temperature and dew point difference by
A. 4
B. 7
C. 6

A

A. 4
Answer (A) is correct. (FAA-H-8083-25B Chap 12) ( ? )
Temperature and dew point in upward moving air converge at a rate of about 4°F (2.20°C) per 1,000 feet. Find the difference between the surface temperature and surface dew point, and then divide by 4°F or 2.20°C as appropriate. The resulting number provides an estimate of the cloud bases in thousands of feet.

178
Q

What information is provided by a Convective Outlook (AC)?
A. It indicates areas of probable convective turbulence and the extent of instability in the upper atmosphere (above 500 mb).
B. It provides prospects of both general and severe thunderstorm activity and specific severe weather threats during the following 8 days.
C. It describes areas of probable severe icing and severe or extreme turbulence during the next 24 hours.

A

B. It provides prospects of both general and severe thunderstorm activity and specific severe weather threats during the following 8 days.
Answer (B) is correct. (AC 00-45H Chap 5) ( ? )
The convective outlook describes the prospects for general and severe thunderstorm activity during the following 8 days. Areas with a high, moderate, enhanced (ENH), marginal (MRGL), or slight risk of severe weather are included.

179
Q

Radiation fog forms
A. always over land.
B. under cloudy skies.
C. almost exclusively near the shore.

A

A. always over land.
Answer (A) is correct. (AC 00-6B Chap 16) ( ? )
Radiation fog is restricted to land because water surfaces cool little from nighttime radiation.

180
Q

Data that may be added (manual weather augmentation) to the Automated Weather Observing System (AWOS) report is limited to
A. the precipitation accumulation report, an automated variable visibility, and wind direction remark.
B. thunderstorms (intensity and direction), precipitation (type and intensity), and obstructions to visibility (dependent on the visibility being 3 miles or less).
C. density altitude, NOTAMs, and reported slant range visibility.

A

B. thunderstorms (intensity and direction), precipitation (type and intensity), and obstructions to visibility (dependent on the visibility being 3 miles or less).
Answer (B) is correct. (AIM Para 7-1-12) ( ? )
Manual weather augmentation remarks added to an AWOS-generated METAR are located in the remarks section. As a general rule, the manual remarks are limited to thunderstorms (intensity and direction), precipitation (type and intensity), and obstructions to vision when the visibility is 3 SM or less.

181
Q

Your flight takes you in the path of a large aircraft. In order to avoid the vortices you should fly
A. at the same altitude as the large aircraft.
B. below the altitude of the large aircraft.
C. above the flight path of the large aircraft.

A

C. above the flight path of the large aircraft.
Answer (C) is correct. (AIM Para 7-3-6, FAA-H-8083-25B) ( ? )
When flying behind a large aircraft, stay at or above the other aircraft’s flight path. Wingtip vortex turbulence tends to sink into the flight path of airplanes operating below the airplane generating the turbulence.

182
Q

In the International Terminal Aerodrome Forecast (TAF), a variable wind direction is noted by “VRB” where the three digit direction usually appears. A calm wind appears in the TAF as
A. 00003KT.
B. VRB00KT
C. 00000KT.

A

C. 00000KT.
Answer (C) is correct. (AC 00-45H Chap 7) ( ? )
In the International Terminal Aerodrome Forecast (TAF), a calm wind appears in the TAF as 00000KT.

183
Q

Which action is recommended regarding an altitude change to get out of jet stream turbulence?
A. Descend if ambient temperature is falling.
B. Descend if ambient temperature is rising.
C. Maintain altitude if ambient temperature is not changing.

A

A. Descend if ambient temperature is falling.
Answer (A) is correct. (AC 00-30B) ( ? )
If turbulence is due to a sloping tropopause, watch the temperature gauge. The point of coldest temperature will be the tropopause penetration. Turbulence will be more pronounced in the temperature-change zone on the stratospheric (upper) side of the sloping tropopause. To get out of this turbulence, descend if the ambient temperature is falling, and climb with a rising temperature.

184
Q
When penetrating a microburst, which aircraft will experience an increase in performance without a change in pitch or power?
A.	3
B.	2
C.	1
See Figure 144
A

C. 1
Answer (C) is correct. (AC 00-54) ( ? )
Position 1 on Fig. 144 indicates the position where the airplane first encounters a headwind and experiences increasing performance. This position is where the headwind component is the greatest, providing an increase in performance without a change in pitch or power.

185
Q

How does the wake turbulence vortex circulate around each wingtip?
A. Inward, upward, and around the wingtip.
B. Outward, upward, and around the wingtip.
C. Counterclockwise when viewed from behind the aircraft.

A

B. Outward, upward, and around the wingtip.
Answer (B) is correct. (AIM Para 7-3-4) ( ? )
Because the pressure differential is caused by a lower pressure above the wing and a higher pressure below the wing, the air from the bottom moves out, up, and around each wingtip.

186
Q

What is corrected altitude (approximate true altitude)?
A. Indicated altitude corrected for temperature variation from standard.
B. Pressure altitude corrected for instrument error.
C. Density altitude corrected for temperature variation from standard.

A

A. Indicated altitude corrected for temperature variation from standard.
Answer (A) is correct. (AC 00-6B Chap 5) ( ? )
Corrected altitude is the indicated altitude of an airplane’s altimeter corrected for temperature variation from standard atmospheric temperature. This is an approximation of true altitude.

187
Q

Forecast winds and temperatures aloft for an international flight may be obtained by consulting
A. the current Ceiling and Visibility Analysis appropriate to the route.
B. Wind and Temperature Aloft Charts prepared by the U.S. National Centers for Environmental Prediction (NCEP).
C. low-level significant weather prog charts published by the departure location host country.

A

B. Wind and Temperature Aloft Charts prepared by the U.S. National Centers for Environmental Prediction (NCEP).
Answer (B) is correct. (AC 00-45H Chap 7) ( ? )
Forecast winds and temperatures aloft for an international flight may be obtained by consulting Winds and Temperatures Aloft Charts prepared by the U.S. National Centers for Environmental Prediction (NCEP).

188
Q

What will be the wind and temperature trend for an SAT ELP TUS flight at 16,000 feet?
A. Windspeed decrease.
B. Wind direction shift from southwest to east.
C. Temperature decrease slightly.
See figure 149

A

B. Wind direction shift from southwest to east.
Answer (B) is correct. (AC 00-45H Chap 7) ( ? )
To determine the wind and temperature trend, the winds and temperatures aloft must be interpolated for 16,000 ft. The table below shows data for 12,000 and 18,000 ft. and the interpolated data for 16,000 ft.

12,000
18,000
16,000
SAT
1911+07
2006-07
2008-02
ELP
0615+08
0113-05
0314-01
TUS
0814+10
0810-05
0811+00
On this route, the wind direction shifts from southwest (200°) to east (080°).
189
Q

Which type clouds may be associated with the jetstream?
A. Cirrostratus cloud band on the polar side and under the jetstream.
B. Cumulonimbus cloud line where the jetstream crosses the cold front.
C. Cirrus clouds on the equatorial side of the jetstream.

A

C. Cirrus clouds on the equatorial side of the jetstream.
Answer (C) is correct. (AC 00-6B Chap 21) ( ? )
Air travels in a “corkscrew” path around the jet stream with upward motion on the equatorial side. Thus, when high-level moisture is available, cirrus clouds form on the equatorial side of the jet stream.

190
Q

En route weather advisories should be obtained from an FSS on
A. 122.2 MHz.
B. 122.1 MHz.
C. 123.6 MHz.

A

A. 122.2 MHz.
Answer (A) is correct. (AC 00-45H Chap 1) ( ? )
Flight Service Stations (FSSs) provide communications capabilities for aircraft flying en route on a common frequency of 122.2 MHz.

191
Q

A thunderstorm first begins when
A. drizzle begins.
B. lifting forces carry cooler air inward and upward.
C. high temperature begins.

A

B. lifting forces carry cooler air inward and upward.
Answer (B) is correct. (FAA-H-8083-25B Chap 12) ( ? )
The cumulus stage is the beginning of the life cycle of a thunderstorm. The key feature of this stage is the updraft. This updraft carries cooler air inward and upward. Growth rate of this stage may exceed 3,000 feet per minute.

192
Q

Downdrafts in thunderstorms can be expected to exceed
A. 2,500 feet per minute.
B. 4,500 feet per minute.
C. 3,500 feet per minute.

A

A. 2,500 feet per minute.
Answer (A) is correct. (AC 00-6A Chap 11) ( ? )
The downdraft signals the mature stage of a thunderstorm. Cold rain in the downdraft retards compressional heating, and the downdraft remains cooler than surrounding air. Therefore, its downward speed is accelerated and may exceed 2,500 feet per minute.

193
Q

The stability of an air mass can usually be determined by
A. the height of the tropopause.
B. cloud types and the type of precipitation.
C. measuring the dry adiabatic lapse rate.

A

B. cloud types and the type of precipitation.
Answer (B) is correct. (FAA-H-8083-25B Chap. 12) ( ? )
Atmospheric stability influences weather by affecting the vertical motion of air. Stable air suppresses vertical motion while unstable air enhances it. Clouds formed in stable air will be shallow and layered, like stratus clouds. Clouds formed in unstable air will have more height and be of the cumulus or cumulonimbus type. Precipitation from stratus clouds tends to be over large areas and lasts for long periods. Precipitation from cumulus clouds tends to be more intense and over short periods.

194
Q

When flying over the top of a severe thunderstorm, the cloud should be overflown by at least
A. 2,500 feet.
B. 1,000 feet for each 10 knots windspeed.
C. 500 feet above any moderate to severe turbulence layer.

A

B. 1,000 feet for each 10 knots windspeed.
Answer (B) is correct. (AC 00-6A Chap 11) ( ? )
The top of a known or suspected severe thunderstorm should be cleared by at least 1,000 ft. of altitude for each 10 kt. of wind speed at the cloud top. In some severe thunderstorms, this clearance may exceed the altitude capability of most aircraft.

195
Q

What happens to residual ice that remains after deice boots are inflated and shed ice?
A. Residual ice decreases with a decrease in airspeed or temperature.
B. Residual ice remains the same until the aircraft exits icing conditions.
C. Residual ice increases with a decrease in airspeed or temperature.

A

C. Residual ice increases with a decrease in airspeed or temperature.
Answer (C) is correct. (AC 91-74B) ( ? )
The FAA recommends that the deicing system be activated at the first indication of icing. Because some residual ice continues to adhere between pneumatic boot system cycles, the wing is never entirely “clean.” The amount of residual ice increases as airspeed and/or temperature decrease due to the more favorable conditions for ice accumulation associated with these conditions. At airspeeds typical of small airplanes, it may take many boot cycles to effectively shed the ice.

196
Q

How can the stability of the atmosphere be determined?
A. Surface temperature/dewpoint spread.
B. Ambient temperature lapse rate.
C. Atmospheric pressure at various levels.

A

B. Ambient temperature lapse rate.
Answer (B) is correct. (AC 00-6B Chap 12) ( ? )
The difference between the ambient temperature lapse rate of a given mass of air and the adiabatic rates of cooling in upward-moving air determines the stability of the air. For example, surface heating or cooling aloft tends to make the air unstable, while surface cooling or warming aloft tends to make the air more stable.

197
Q

The threshold windspeed in the jet stream for clear air turbulence is generally considered to be
A. 100 knots.
B. 120 knots.
C. 110 knots.

A

C. 110 knots.
Answer (C) is correct. (AC 00-30C) ( ? )
The threshold windspeed in the jet stream for clear air turbulence (CAT) is considered to be 110 knots. Windspeed in jet streams can be much stronger than 110 knots, and the probability of encountering CAT increases proportionally with the windspeed and the wind shear it generates. It is not the wind speed itself that causes CAT. It is the wind shear, or difference in windspeed from one level or point to another, that causes the wave motion or overturning in the atmosphere that is turbulence to an aircraft.

198
Q

How are haze layers cleared or dispersed?
A. By wind or the movement of air.
B. By evaporation similar to the clearing of fog.
C. By convective mixing in cool night air.

A

A. By wind or the movement of air.
Answer (A) is correct. (AC 00-6B Chap 16) ( ? )
Haze layers may be cleared or dispersed by the wind, or heating during the day may cause convective movement of the air, spreading the haze to a higher altitude.

199
Q

The tropopause varies in height from
A. 20,000 feet or below at the poles to 65,000 feet at the equator.
B. 15,000 feet or below at the poles to 70,000 feet at the equator.
C. 32,000 feet or below at the poles to 320,000 feet at the equator.

A

A. 20,000 feet or below at the poles to 65,000 feet at the equator.
Answer (A) is correct. (AC 00-6A Chap 13) ( ? )
The height of the tropopause varies in height from about 20,000 feet over the poles to about 65,000 feet over the equator. It is higher in the summer than in winter.

200
Q

Clear air turbulence (CAT) can be found
A. inside convective clouds.
B. outside convective clouds.
C. near local terrain features.

A

B. outside convective clouds.
Answer (B) is correct. (AC 00-30C) ( ? )
Clear air turbulence (CAT) is best defined as turbulence encountered outside of convective clouds. The term CAT is used to describe the turbulence within and in the vicinity of cirrus and standing lenticular clouds.

201
Q

Advection fog usually deepens in winds up to about
A. 10 knots.
B. 15 knots.
C. 5 knots.

A

B. 15 knots.
Answer (B) is correct. (AC 00-6B Chap 16) ( ? )
Advection fog is most common along coastal areas but can also develop deep in continental areas. Advection fog deepens as wind speed increases up to about 15 knots.

202
Q

What type turbulence should be reported when it causes changes in altitude and/or attitude more than two-thirds of the time, with the aircraft remaining in positive control at all times?
A. Continuous moderate turbulence.
B. Intermittent moderate turbulence.
C. Continuous severe chop.

A

A. Continuous moderate turbulence.
Answer (A) is correct. (AIM Para 7-1-25) ( ? )
Moderate turbulence is defined as turbulence that causes changes in altitude and/or attitude, but the aircraft remains in positive control at all times. Continuous is defined as occurring more than two-thirds of the time.

203
Q

The following weather condition may be conducive to severe in-flight icing:
A. visible moisture at temperatures below 5°C ambient temperature.
B. visible rain at temperatures below 10°C ambient temperature.
C. visible rain at temperatures below 0°C ambient air temperature.

A

C. visible rain at temperatures below 0°C ambient air temperature.
Answer (C) is correct. (AC 91-74B) ( ? )
Visible rain at ambient temperatures below 0°C (32°F) indicates a condition that is conducive to severe in-flight icing.

204
Q

Which atmospheric factor causes rapid movement of surface fronts?
A. Upper low located directly over the surface low.
B. Upper winds blowing across the front.
C. The cold front overtaking and lifting the warm front.

A

B. Upper winds blowing across the front.
Answer (B) is correct. (AC 00-6B Chap 7) ( ? )
The upper wind flow dictates to a great extent the movement of the front. Systems tend to move with the upper winds. When winds aloft blow across a front, it tends to move with the wind.

205
Q

At what time are current AIRMETs broadcast in their entirety by the Hazardous Inflight Weather Advisory Service (HIWAS)?
A. Every 15 minutes until the AIRMET is canceled.
B. There is a continuous broadcast over selected VORs of Inflight Weather Advisories.
C. 15 and 45 minutes after the hour during the first hour after issuance, and upon receipt.

A

B. There is a continuous broadcast over selected VORs of Inflight Weather Advisories.
Answer (B) is correct. (AC 00-45H Chap 1) ( ? )
HIWAS is a continuous broadcast over selected VORs of in-flight weather advisories including AIRMETs, SIGMETs, convective SIGMETs, center weather advisories, urgent PIREPs, and summarized severe weather forecast alerts.

206
Q

In the mid-latitudes, wind speed in the jet stream averages considerably less in the
A. spring.
B. summer.
C. winter.

A

B. summer.
Answer (B) is correct. (AC 00-6B Chap 8) ( ? )
In mid-latitudes, wind speed in the jet stream averages considerably stronger in winter than in summer.

207
Q
What effect will a microburst encounter have upon the aircraft in position 3?
A.	Decreasing headwind.
B.	Increasing tailwind.
C.	Strong downdraft.
See figure 144
A

C. Strong downdraft.
Answer (C) is correct. (AC 00-54) ( ? )
At position 3 in Fig. 144, the airplane is experiencing a strong downdraft as it is approaching the center of the microburst.

208
Q

Which type of weather can only be directly observed during flight and then reported in a PIREP?
A. Jetstream-type winds and icing.
B. Turbulence and structural icing.
C. Level of the tropopause and turbulence.

A

B. Turbulence and structural icing.
Answer (B) is correct. (AC 00-45H Chap 3) ( ? )
Pilots are urged to volunteer directly observed reports of thunderstorms, structural icing, turbulence, wind shear, cloud bases (tops and layers), visibility, precipitation, wind, and temperature at altitude.

209
Q

What time period is covered by the outlook section of the Convective SIGMET?
A. No more than 2 hours after the valid time.
B. 24 hours after the valid time.
C. 2 to 6 hours after the valid time.
See figure 148

A

C. 2 to 6 hours after the valid time.
Answer (C) is correct. (AIM Para 7-1-7) ( ? )
The outlook for a particular region is appended to the convective SIGMET. The outlook is a forecast and meteorological discussion for thunderstorm systems that are expected to require convective SIGMET issuances during a period 2 to 6 hr. after the valid time. Note that on Fig. 148 the outlook is good until 2355Z, which is 4 hr. after the valid time of 1955Z.

210
Q

Which is a necessary condition for the occurrence of a low-level temperature inversion wind shear?
A. The temperature differential between the cold and warm layers must be at least 10°C.
B. A calm or light wind near the surface and a relatively strong wind just above the inversion.
C. A wind direction difference of at least 30° between the wind near the surface and the wind just above the inversion.

A

B. A calm or light wind near the surface and a relatively strong wind just above the inversion.
Answer (B) is correct. (AC 00-6B Chap 17) ( ? )
A temperature inversion forms near the surface on a clear night with calm or light surface wind, and a relatively strong wind just above the inversion is necessary to form a low-level temperature inversion wind shear.

211
Q

A SIGMET is issued when weather affects
A. 5,000 square miles or more.
B. 1,000 square miles or more.
C. 3,000 square miles or more.

A

C. 3,000 square miles or more.
Answer (C) is correct. (AC 00-45H Chap 5) ( ? )
A SIGMET may be issued in the contiguous U.S. when certain conditions are affecting or, in the judgment of the forecaster, are expected to affect an area of at least 3,000 square miles of an area judged to have a significant impact on the safety of aircraft operations.

212
Q

Which type precipitation is an indication that supercooled water is present?
A. Freezing rain.
B. Ice pellets.
C. Wet snow.

A

A. Freezing rain.
Answer (A) is correct. (AC 91-74B) ( ? )
Rain falling through colder air may become supercooled, freezing on impact as freezing rain. Supercooled water is liquid water that is colder than 0°C, and its impact on an object induces freezing.

213
Q

A cyclone is similar to a tropical storm in
A. the northern Pacific with winds up to 64 knots.
B. the north Atlantic with winds 35 to 64 knots.
C. the Indian Ocean with winds 65 knots or more.

A

C. the Indian Ocean with winds 65 knots or more.
Answer (C) is correct. (AC 00-6B Chap 21) ( ? )
A strong tropical cyclone (with winds of 65 knots or more) is known in the Indian Ocean as a “cyclone.”

214
Q

The VV001 in the following METAR indicates
METAR KFSM 131756Z AUTO 00000KT M1/4SM R25/0600V1000FT -RA FG VV001 A2989 RMK AO2 VIS 3/4 RWY19 CHINO RWY19 $.

A. the variability value is 100 feet.
B. a 100-foot indefinite ceiling.
C. an observer reported the vertical visibility as 100 feet.

A

B. a 100-foot indefinite ceiling.
Answer (B) is correct. (AC 00-45H Chap 3) ( ? )
The sky condition in the METAR is shown as VV001, which means a vertical visibility (spoken as an indefinite ceiling) of 100 ft.

215
Q

The tropopause is generally found when the free air temperatures are
A. colder than -60°C.
B. between -40° and -55°C.
C. between -55° and -65°C.

A

C. between -55° and -65°C.
Answer (C) is correct. (AC 00-6B Chap 1) ( ? )
In a standard atmosphere, the tropopause has a temperature of -56.5°C and a height of about 36,000 feet above the Earth. While both the temperature and altitude ranges of the tropopause are quite large, this answer choice is the best answer to the question given its inclusion of the standard temperature for the tropopause.

216
Q
What approximate wind direction, speed, and temperature (relative to ISA) are expected for a flight over OKC at FL 370?
A.	265° true; 27 knots; ISA +1°C.
B.	260° true; 27 knots; ISA +6°C.
C.	260° magnetic; 27 knots; ISA +10°C.
See figure 149
A
B.	260° true; 27 knots; ISA +6°C.
Answer (B) is correct. (AC 00-45H Chap 7) ( ? )
For conditions at FL 370 over OKC in Fig. 149, interpolate between values at FL 340 and FL 390. First decode the two given flight levels:
FL 340
=
250°
at
27 kt.
and
–43°C
FL 390
=
270°
at
27 kt.
and
–54°C
Difference
=
20°
at
0 kt.
and
–11°C
Interpolation for each value gives:
FL 370 = 262° at 27 kt. and –50°C
Since the FB is rounded to the nearest 10°, the wind direction is 260°. To compare the temperature to standard temperature, remember that the temperature in the tropopause (36,000 ft. MSL and above) is approximately –56°C. Thus, –50°C is ISA +6°C. (ISA is the International Standard Atmosphere. ISA temperature at sea level is 15°C and decreases at a rate of 2°/1,000 ft. up to 36,000 ft. MSL.)
217
Q

What sources reflect the most accurate information on current and forecast icing conditions?
A. PIREP’s, Area Forecast, and the Freezing Level Chart.
B. Low-Level Sig Weather Prog Chart, RADAT’s, and the Area Forecast.
C. PIREP’s, AIRMET Zulu’s, and SIGMET’s.

A

C. PIREP’s, AIRMET Zulu’s, and SIGMET’s.
Answer (C) is correct. (AC 91-74B) ( ? )
Pilot reports (PIREPs) can reflect the most current icing conditions for a specific area, including type and intensity. AIRMET Zulu is for icing and freezing levels for a specified time and normally an outlook. A SIGMET is issued for areas of severe icing.

218
Q

What is an important characteristic of wind shear?
A. It may be associated with either a wind shift or a windspeed gradient at any level in the atmosphere.
B. It usually exists only in the vicinity of thunderstorms, but may be found near a strong temperature inversion.
C. It is primarily associated with the lateral vortices generated by thunderstorms.

A

A. It may be associated with either a wind shift or a windspeed gradient at any level in the atmosphere.
Answer (A) is correct. (AC 00-6B Chap 17) ( ? )
Wind shear is the rate of change of wind velocity. The differences may be in wind speed, wind direction, or both. An important characteristic is that wind shear may be associated with either a wind shift or a wind speed gradient at any level in the atmosphere.

219
Q

When thrust is being used to manage the glide slope, which characteristics should be observed when a headwind shears to a constant tailwind?
A. Increase in pitch, decrease in vertical speed, and loss of airspeed.
B. Decrease in pitch, decrease in vertical speed, and increase in airspeed.
C. Decrease in pitch, increase in vertical speed, and loss of airspeed followed by on speed indications.

A

C. Decrease in pitch, increase in vertical speed, and loss of airspeed followed by on speed indications.
Answer (C) is correct. (AC 00-54) ( ? )
In contrast to shears that increase airspeed, an increasing tailwind (or decreasing headwind) shear will decrease indicated airspeed and performance capability. Due to airspeed loss, the airplane may tend to pitch down to regain trim speed. Be prepared!

220
Q

For international flights, a U.S. High-Level Significant Weather Prognostic Chart is prepared for use
A. at any altitude above 29,000.
B. between 25,000 feet and 63,000 feet pressure altitude.
C. between FL 180 and FL 630

A

B. between 25,000 feet and 63,000 feet pressure altitude.
Answer (B) is correct. (AC 00-45H Chap 8) ( ? )
Numerous High-Level Significant Weather Prognostic Charts are produced by the Aviation Weather Center (AWC) for international flights. They are prepared for flights between 25,000 ft. and 63,000 ft. pressure altitude (or FL 250 to FL 630).

221
Q

SIGMETs may be issued for the contiguous U.S.
A. as convective SIGMETs when obscured thunderstorms are forecast.
B. and as convective SIGMETs for Alaska when obscured thunderstorms are forecast.
C. and as convective SIGMETs for Alaska and Hawaii when squall lines are forecast.

A

A. as convective SIGMETs when obscured thunderstorms are forecast.
Answer (A) is correct. (AC 00-45H) ( ? )
Convective SIGMETs are issued for the conterminous U.S. (CONUS) instead of SIGMETs for thunderstorms. Any Convective SIGMET implies severe or greater turbulence, severe icing, and low-level wind shear. This includes the 48 states within the contiguous U.S.

222
Q

What condition is indicated when ice pellets are encountered during flight?
A. Snow at higher levels.
B. Freezing rain at higher levels.
C. Thunderstorms at higher levels.

A

B. Freezing rain at higher levels.
Answer (B) is correct. (FAA-H-8083-25B Chap 12) ( ? )
Ice pellets always indicate freezing rain at higher altitudes. Ice pellets are formed when rain freezes during descent.

223
Q

A jet stream is a river like flow of high-altitude wind following the planetary atmospheric wave patterns with speeds of
A. 50 knots or greater.
B. 100 knots or greater.
C. 71 knots or greater.

A

A. 50 knots or greater.
Answer (A) is correct. (AC 00-6A Chap 13) ( ? )
To be classified as a jet stream, the concentrated winds must be 50 knots or greater.

224
Q

At lower levels of the atmosphere, friction causes the wind to flow across isobars into a low because the friction
A. decreases pressure gradient force.
B. decreases windspeed and Coriolis force.
C. creates air turbulence and raises atmospheric pressure.

A

B. decreases windspeed and Coriolis force.
Answer (B) is correct. (AC 00-6B Chap 7) ( ? )
Surface friction decreases the wind speed and Coriolis force but does not affect pressure gradient force. The stronger pressure gradient turns the wind across the isobars toward lower pressure.

225
Q

During the life cycle of a thunderstorm, which stage is characterized predominately by downdrafts?
A. Dissipating.
B. Cumulus.
C. Mature.

A

A. Dissipating.
Answer (A) is correct. (AC 00-6B Chap 19) ( ? )
Thunderstorms have three life cycles: cumulus, mature, and dissipating. In the dissipating stage, the storm is characterized by downdrafts as the storm rains itself out.

226
Q

What is the result when water vapor changes to the liquid state while being lifted in a thunderstorm?
A. Latent heat is absorbed from the surrounding air by the water droplet.
B. Latent heat is transformed into pure energy.
C. Latent heat is released to the atmosphere.

A

C. Latent heat is released to the atmosphere.
Answer (C) is correct. (AC 00-6B Chap 3) ( ? )
In a thunderstorm, the updrafts (lifting action) result in water vapor changing to liquid or condensation. Condensation releases latent heat to the atmosphere, which partially offsets cooling in the saturated updraft and increases buoyancy within the cloud.

227
Q

Significant precipitation usually requires clouds to be
A. 3,000 feet thick.
B. 4,000 feet thick.
C. 2,000 feet thick.

A

B. 4,000 feet thick.
Answer (B) is correct. (AC 00-6B Chap 14) ( ? )
To produce significant precipitation, clouds usually are 4,000 feet thick or more.

228
Q

What is the general direction of movement of a hurricane located in the Caribbean or Gulf of Mexico region?
A. Northwesterly curving to northeasterly.
B. Westerly, until encountering land, then easterly.
C. Counterclockwise over open water, then dissipating outward over land.

A

A. Northwesterly curving to northeasterly.
Answer (A) is correct. (AC 00-6B Chap 21) ( ? )
Hurricanes located in the Caribbean or Gulf of Mexico move in a direction between west and northwest while in low latitudes. As the hurricane moves toward the midlatitudes, the prevailing westerlies will gain control, and the hurricane will curve to the northeast.

229
Q

What condition is reported at Dallas (KDAL)?
A. Temperature/dewpoint spread is 8°F.
B. The tops of the overcast is 10,000 feet.
C. Altimeter setting is 30.07.
See figure 145

A

C. Altimeter setting is 30.07.
Answer (C) is correct. (AC 00-45H Chap 3) ( ? )
Altimeter settings are reported in a four-digit format in inches of mercury prefixed with an “A” to denote the units of pressure. KDAL altimeter setting is coded as A3007, which means 30.07 inches of mercury.

230
Q

Freezing rain encountered during climb is normally evidence that
A. ice pellets at higher altitudes have changed to rain in the warmer air below.
B. a climb can be made to a higher altitude without encountering more than light icing.
C. a layer of warmer air exists above.

A

C. a layer of warmer air exists above.
Answer (C) is correct. (AC 00-6B Chap 14) ( ? )
Freezing rain is rain that falls through colder air and becomes supercooled, freezing on impact. If encountered, it means that there is a layer of warmer air above.

231
Q

Dew point fronts result from
A. air density due to temperatures.
B. air density differences due to humidity levels.
C. temperatures aloft.

A

B. air density differences due to humidity levels.
Answer (B) is correct. (AC 00-6A Chap 8) ( ? )
During a considerable part of the year, dew point fronts are common in western Texas and New Mexico northward over the plains states. Moist air flows north from the Gulf of Mexico and abuts the dryer and, therefore, slightly denser air flowing from the southwest. It is the moisture (humidity) difference that causes the air density differences.

232
Q
What effect will a microburst encounter have upon the aircraft in position 4?
A.	Significant performance increase.
B.	Strong tailwind.
C.	Strong updraft.
See figure 144
A

B. Strong tailwind.
Answer (B) is correct. (AC 00-54) ( ? )AC | Advisory Circular
At position 4 in Fig. 144, the airplane is encountering a strong tailwind in addition to a strong downdraft and a decrease in airplane performance.

233
Q

Which event usually occurs after an aircraft passes through a front into the colder air?
A. Atmospheric pressure increases.
B. Wind direction shifts to the left.
C. Temperature/dewpoint spread decreases.

A

A. Atmospheric pressure increases.
Answer (A) is correct. (FAA-H-8083-25B Chap 12) ( ? )
A front lies in a pressure trough, and pressure generally is higher in cold air. Thus, when you cross a front directly into colder air, barometric pressure usually increases.

234
Q

Interpret the path of the jetstream.
A. Northern California coast, over Utah, turns southerly to the Gulf of Mexico, turning northeast over Florida.
B. Oregon, Idaho, Wyoming, Nebraska, Iowa, and across the Great Lakes.
C. The Alaska area, across Canada to Montana, South Dakota, then across the Great Lakes area.
See figures 153,154,155

A

A. Northern California coast, over Utah, turns southerly to the Gulf of Mexico, turning northeast over Florida.
Answer (A) is correct. (AC 00-45H Chap 5) ( ? )
To interpret the path of the jet stream, compare the isotachs on the 300- and 200-mb charts. The jet stream has winds of 50 kt. or greater. These greater wind speeds are found from the central California coast to Utah, then southward to the Gulf, returning across Florida and then northeastward.

235
Q

Which weather condition is defined as an anticyclone?
A. Calm.
B. COL.
C. High pressure area.

A

C. High pressure area.
Answer (C) is correct. (AC 00-6B Chap 5) ( ? )
An anticyclone is an area of high atmospheric pressure. The circulation is clockwise in the Northern Hemisphere, counterclockwise in the Southern Hemisphere, and undefined at the Equator.

236
Q

You can expect turbulence when winds exceed
A. 30 knots downslope in mountains.
B. 30 knots upslope in mountains.
C. 40 knots across the mountains.

A

C. 40 knots across the mountains.
Answer (C) is correct. (AC 00-6A Chap 9) ( ? )
When flying over rugged hills or mountains, you may encounter severe turbulence. Anticipate turbulence when wind speed across mountains exceeds about 40 knots.

237
Q

Which are the only cloud types forecast in the Terminal Aerodrome Forecast?
A. Stratocumulus.
B. Altocumulus.
C. Cumulonimbus.

A

C. Cumulonimbus.
Answer (C) is correct. (AC 00-45H Chap 7) ( ? )
If cumulonimbus clouds are expected at the airport, the contraction CB is appended to the cloud layer that represents the base of the cumulonimbus cloud(s). Cumulonimbus clouds are the only cloud type forecast in the TAF.

238
Q

Intense radar storm echos should be avoided by a margin of
A. 25 NM.
B. 30 NM.
C. 20 NM.

A

C. 20 NM.
Answer (C) is correct. (AC 00-24C) ( ? )
The most intense echos are severe thunderstorms. Remember that hail may fall several miles from the cloud, and hazardous turbulence may extend as much as 20 NM from the cloud. Avoid the most intense echos by at least 20 NM.

239
Q

Which airplane performance characteristics should be recognized during takeoff when encountering a tailwind shear that increases in intensity?
A. Decreased takeoff distance.
B. Loss of, or diminished, airspeed performance.
C. Increased climb performance immediately after takeoff.

A

B. Loss of, or diminished, airspeed performance.
Answer (B) is correct. (AC 00-54) ( ? )
As a tailwind shear increases in intensity, the aircraft performance decreases. More power will be required to counter the decrease in airspeed. As the tailwind intensity increases, there will be a loss of, or diminished, airspeed performance.

240
Q

What is indicated by the term “embedded thunderstorms”?
A. Severe thunderstorms are embedded in a squall line.
B. Thunderstorms are predicted to develop in a stable air mass.
C. Thunderstorms are obscured by other types of clouds.

A

C. Thunderstorms are obscured by other types of clouds.
Answer (C) is correct. (FAA-H-8083-25B Chap 12) ( ? )
The term “embedded thunderstorms” means that the thunderstorms are embedded in clouds or thick haze layers and cannot be seen.

241
Q

Whiteout conditions occur when
A. light is reflected back and forth countless times between snow and clouds, eliminating all shadows.
B. the low angle of the sun’s rays strike the snow surface and eliminate shadows to the eye due to the “prism effect.”
C. snow eliminates the color contrast of the surface and clouds during daytime.

A

A. light is reflected back and forth countless times between snow and clouds, eliminating all shadows.
Answer (A) is correct. (AC 00-6B Chap 22) ( ? )
“Whiteout” is a visibility restricting phenomenon that occurs in the Arctic when a layer of cloudiness of uniform thickness overlies a snow or ice covered surface. Parallel rays of the sun are broken up and diffused when passing through the cloud layer so that they strike the snow surface from many angles. The diffused light then reflects back and forth countless times between the snow and the cloud, eliminating all shadows.

242
Q

An aircraft that encounters a headwind of 45 knots, within a microburst, may expect a total shear across the microburst of
A. 90 knots.
B. 40 knots.
C. 80 knots.

A

A. 90 knots.
Answer (A) is correct. (AIM Para 7-1-26) ( ? )
If a headwind in a microburst is 45 kt., the wind will be going in the opposite direction on the other side of the microburst at presumably the same 45 kt., resulting in a wind shear between the headwind and tailwind of 90 kt.

243
Q

What method was used to obtain the METAR at Tyler (KTYR) at 1753Z?
A. Automated Weather Observing System (AWOS), without a precipitation discriminator.
B. Automated Surface Observing System (ASOS), having a precipitation discriminator.
C. Automatic Meteorological Observing Station (AMOS), with a precipitation discriminator.
See figure 146

A

B. Automated Surface Observing System (ASOS), having a precipitation discriminator.
Answer (B) is correct. (AIM Para 7-1-12) ( ? )
After the date/time group on the METAR for KTYR is the contraction AUTO, which means that the METAR is a fully automated report. Currently, an ASOS is the only automated station that performs the basic observing functions necessary to generate a METAR report. Additionally, the remark A02 means the automated station has a precipitation discriminator.

244
Q

Where is the normal location of the jetstream relative to surface lows and fronts?
A. The jetstream is located over the low and crosses both the warm front and the cold front.
B. The jetstream is located south of the low and warm front.
C. The jetstream is located north of the surface systems.

A

C. The jetstream is located north of the surface systems.
Answer (C) is correct. (AC 00-6B Chap 8) ( ? )
Strong, long-trajectory jet streams usually are associated with well-developed surface lows and frontal systems beneath upper troughs or lows. The jet stream is to the north as a surface low develops, and the low moves nearer the jet stream as the low deepens. The occluding low moves north of the jet stream, and the jet stream crosses the frontal system near the point of occlusion.

245
Q

Which is a definition of “severe wind shear”?
A. Any rapid change in wind direction or velocity which causes airspeed changes greater than 15 knots or vertical speed changes greater than 500 ft/min.
B. Any change of airspeed greater than 20 knots which is sustained for more than 20 seconds or vertical speed changes in excess of 100 ft/min.
C. Any rapid change of horizontal wind shear in excess of 25 knots; vertical shear excepted.

A

A. Any rapid change in wind direction or velocity which causes airspeed changes greater than 15 knots or vertical speed changes greater than 500 ft/min.
Answer (A) is correct. (AC 00-54) ( ? )
A severe wind shear is defined as a rapid change in wind direction or velocity causing airspeed changes greater than 15 kt. or vertical speed changes greater than 500 ft/min.

246
Q

How does Coriolis force affect wind direction in the Southern Hemisphere?
A. Causes wind to flow out of a low toward a high.
B. Causes clockwise rotation around a low.
C. Has exactly the same effect as in the Northern Hemisphere.

A

B. Causes clockwise rotation around a low.
Answer (B) is correct. (FAA-H-8083-25B Chap 12) ( ? )
Coriolis force deflects air to the left in the Southern Hemisphere, which is the opposite from the Northern Hemisphere. In the Southern Hemisphere, the wind blows clockwise around a low.

247
Q

Cloud formation is a process of
A. stagnant cooling.
B. advective cooling.
C. expansional cooling.

A

C. expansional cooling.
Answer (C) is correct. (AC 00-6B Chap 11) ( ? )
Of the three basic processes that cool air to saturation, expansional cooling is the major cause of cloud formation.

248
Q

When does minimum temperature normally occur during a 24-hour period?
A. About 1 hour before sunrise.
B. At midnight.
C. After sunrise.

A

C. After sunrise.
Answer (C) is correct. (AC 00-6B Chap 4) ( ? )
Diurnal variation is the change in temperature within a 24-hr. period. The minimum temperature normally occurs after sunrise, sometimes as much as 1 hr. after.

249
Q

A clear area in a line of thunderstorm echoes on a radar scope indicates
A. an area where precipitation drops are not detected.
B. an area of no convective turbulence.
C. the absence of clouds in the area.

A

A. an area where precipitation drops are not detected.
Answer (A) is correct. (AC 00-24C) ( ? )
Weather radar detects droplets of precipitation. If a radarscope displays a clear area, it indicates an area where no precipitation drops are detected.

250
Q

What effect would a light crosswind have on the wingtip vortices generated by a large airplane that has just taken off?
A. The downwind vortex will tend to remain on the runway longer than the upwind vortex.
B. The upwind vortex will tend to remain on the runway longer than the downwind vortex.
C. A crosswind will rapidly dissipate the strength of both vortices.

A

B. The upwind vortex will tend to remain on the runway longer than the downwind vortex.
Answer (B) is correct. (AIM Para 7-3-4) ( ? )
When vortices of large aircraft sink close to the ground (within about 200 ft.), they tend to move laterally over the ground at a speed of about 2 or 3 kt. in the direction of each generating wing. Thus, the downwind vortex on a crosswind is blown away quickly, but the upwind vortex remains on the runway longer.

251
Q

Which primary source contains information regarding the expected weather at the destination airport at the ETA?
A. Weather Depiction Charts.
B. Terminal Aerodrome Forecast.
C. Low-Level Prog Chart.

A

B. Terminal Aerodrome Forecast.
Answer (B) is correct. (AC 00-45H Chap 7) ( ? )
A Terminal Aerodrome Forecast (TAF) is a concise statement of the expected meteorological conditions at an airport during a specified period (usually 24 hr.). Thus, a TAF contains information regarding the expected weather at the destination airport at the ETA.

252
Q

What is a feature of a stationary front?
A. Weather conditions are a combination of strong cold front and strong warm front weather.
B. Surface winds tend to flow parallel to the frontal zone.
C. The warm front surface moves about half the speed of the cold front surface.

A

B. Surface winds tend to flow parallel to the frontal zone.
Answer (B) is correct. (AC 00-6B Chap 7) ( ? )
The opposing forces exerted by adjacent air masses of different densities are such that the frontal surface between them shows little or no movement and a stationary front results. The surface winds tend to flow parallel to this frontal zone.

253
Q

What will be the wind and temperature trend for a DSM-LIT-SHV flight at 12,000 feet?
A. Temperature decrease.
B. Windspeed decrease.
C. Wind direction shift from northwest to southeast.
See figure 149

A

B. Windspeed decrease.
Answer (B) is correct. (AC 00-45H Chap 7) ( ? )
In determining the wind and temperature trend, it is easier to view in a table format. Note that, in this question, no interpolation is needed since 12,000 ft. is an altitude at which data are supplied from the Winds and Temperatures Aloft Forecast. Below is a table of the data:

DSM
LIT
SHV
12,000 ft.
3022+00
2808+06
2106+06
During this flight, the wind speed decreases from 22 kt. to 8 kt. to 6 kt.
254
Q

Vertical wind shear can be determined by comparing winds on vertically adjacent constant pressure charts. The vertical wind shear that is critical for probability of turbulence is
A. 5 knots or more per 1,000 feet.
B. greater than 8 knots per 1,000 feet.
C. 4 knots or greater per 1,000 feet.

A

A. 5 knots or more per 1,000 feet.
Answer (A) is correct. (AC 00-30C) ( ? )
The vertical wind shear critical for probable turbulence is 5 kt. or more per 1,000 ft.

255
Q

Wingtip vortices created by large aircraft tend to
A. accumulate and remain for a period of time at the point where the takeoff roll began.
B. sink below the aircraft generating the turbulence.
C. rise from the surface to traffic pattern altitude.

A

B. sink below the aircraft generating the turbulence.
Answer (B) is correct. (AIM Para 7-3-4) ( ? )
The vortices created by large aircraft sink at a rate of several hundred feet per minute, slowing their descent and diminishing in strength with time and distance behind the generating aircraft. Pilots should fly at or above the preceding aircraft’s flight path, avoiding the area behind and below the vortex-generating aircraft.

256
Q

What action is appropriate when encountering the first ripple of reported clear air turbulence (CAT)?
A. Adjust airspeed to that recommended for rough air.
B. Extend gear to provide more drag and increase stability.
C. Extend flaps to decrease wind loading.

A

A. Adjust airspeed to that recommended for rough air.
Answer (A) is correct. (FAA-H-8083-25B Chap 5) ( ? )
When encountering the first ripple of clear air turbulence (CAT), it is appropriate to reduce airspeed to that recommended for rough air. This action decreases the amount of stress that is put on the airplane while in turbulence.

257
Q

What wind condition prolongs the hazards of wake turbulence on a landing runway for the longest period of time?
A. Light quartering headwind.
B. Direct tailwind.
C. Light quartering tailwind.

A

C. Light quartering tailwind.
Answer (C) is correct. (AIM Para 7-3-4) ( ? )
Light quartering tailwinds require the maximum caution because they can move the vortices of preceding aircraft forward into the touchdown zone and hold the upwind vortex on the runway.

258
Q

Which wind-shear condition results in a loss of airspeed?
A. Decreasing headwind and increasing tailwind.
B. Decreasing headwind or tailwind.
C. Increasing headwind and decreasing tailwind.

A

A. Decreasing headwind and increasing tailwind.
Answer (A) is correct. (AC 00-54) ( ? )
When the headwind component decreases or the tailwind component increases, initially the airspeed decreases, the aircraft pitches down, and the altitude decreases.

259
Q

What is the expected duration of an individual microburst?
A. Five minutes with maximum winds lasting approximately 2 to 4 minutes.
B. Seldom longer than 15 minutes from the time the burst strikes the ground until dissipation.
C. One microburst may continue for as long as an hour.

A

B. Seldom longer than 15 minutes from the time the burst strikes the ground until dissipation.
Answer (B) is correct. (AIM Para 7-1-26) ( ? )
An individual microburst will seldom last longer than 15 min. from the time it strikes the ground until dissipation. The horizontal winds continue to increase during the first 5 min., with maximum-intensity winds lasting approximately 2 to 4 min.

260
Q

What is the expected duration of an individual microburst?
A. Seldom longer than 15 minutes from the time the burst strikes the ground until dissipation.
B. One microburst may continue for as long as 2 to 4 hours.
C. Two minutes with maximum winds lasting approximately 1 minute.

A

A. Seldom longer than 15 minutes from the time the burst strikes the ground until dissipation.
Answer (A) is correct. (AIM Para 7-1-26) ( ? )
An individual microburst will seldom last longer than 15 min. from the time it strikes ground until dissipation. The horizontal winds continue to increase during the first 5 min., with maximum-intensity winds lasting approximately 2 to 4 min.

261
Q

What information from the control tower is indicated by the following transmission?
“SOUTH BOUNDARY WIND ONE SIX ZERO AT TWO FIVE, WEST BOUNDARY WIND TWO FOUR ZERO AT THREE FIVE.”

A. Wake turbulence exists on the west side of the active runway.
B. There is a possibility of wind shear over or near the airport.
C. A downburst is located at the center of the airport.

A

B. There is a possibility of wind shear over or near the airport.
Answer (B) is correct. (AIM Para 4-3-7) ( ? )
The information for the transmission was provided by the Low-Level Wind Shear Alert System (LLWAS). This system detects the possibility of low-level wind shear over or near the airport. The system compares the wind measured around the airport with the wind measured at the center field location. When conditions exist, the tower controller will provide the site’s location and wind. Note that the transmission in the question indicates wind from different directions at the airport.

262
Q

Which process causes adiabatic cooling?
A. Expansion of air as it rises.
B. Release of latent heat during the vaporization process.
C. Movement of air over a colder surface.

A

A. Expansion of air as it rises.
Answer (A) is correct. (AC 00-6B Chap 11) ( ? )
Adiabatic cooling is caused by the expansion of air as it rises. Any time air is moved upward, it expands because of decreasing atmospheric pressure. When air expands, it cools.

263
Q

What is a likely location of clear air turbulences?
A. In an upper trough on the polar side of a jet stream.
B. Near a ridge aloft on the equatorial side of a high-pressure flow.
C. Downstream of the equatorial side of a jet stream.

A

A. In an upper trough on the polar side of a jet stream.
Answer (A) is correct. (AC 00-30C) ( ? )
A preferred location of clear air turbulence is in an upper trough on the polar side of the jet stream. Cold and warm advection along with strong wind shears develop near the jet stream, especially where the curvature of the jet stream sharply increases in deepening upper troughs.

264
Q

What airport condition is reported by the tower when more than one wind condition at different positions on the airport is reported?
A. Light and variable.
B. Wind shear.
C. Frontal passage.

A

B. Wind shear.
Answer (B) is correct. (AIM Para 4-3-7) ( ? )
When the tower reports more than one wind condition at different positions on the airport, it means that wind shear is probable. Various sensors of the Low-Level Wind Shear Alert System measure the wind, and when differences become excessive, the tower will provide more than one wind condition at different positions.

265
Q

Which type cloud is associated with violent turbulence and a tendency toward the production of funnel clouds?
A. Cumulonimbus mamma.
B. Stratocumulus.
C. Standing lenticular.

A

A. Cumulonimbus mamma.
Answer (A) is correct. (AC 00-6B Chap 13) ( ? )
Cumulonimbus mamma is a cumulonimbus cloud having hanging protuberances like pouches or udders on the underside of the cloud, usually indicative of severe turbulence. This type of cloud can produce a tornado.

266
Q

Which statement is true concerning the wake turbulence produced by a large transport aircraft?
A. Wake turbulence behind a propeller-driven aircraft is negligible because jet engine thrust is a necessary factor in the formation of vortices.
B. The vortex characteristics of any given aircraft may be altered by extending the flaps or changing the speed.
C. Vortices can be avoided by flying 300 feet below and behind the flight path of the generating aircraft.

A

B. The vortex characteristics of any given aircraft may be altered by extending the flaps or changing the speed.
Answer (B) is correct. (AIM Para 7-3-3) ( ? )
The strength of the wake turbulence produced by a large transport aircraft is governed by the weight, speed, and shape of the wing of the generating aircraft. The vortex characteristics of any given aircraft may be altered by the extension of flaps or other wing-configuring devices, as well as a change in speed.

267
Q

Clouds with extensive vertical development over mountainous terrain are a sign of
A. a dry adiabatic lapse rate.
B. a stable air mass.
C. an unstable air mass.

A

C. an unstable air mass.
Answer (C) is correct. (AC 00-6B) ( ? )
Winds across mountains cause mountain waves that are associated with severe turbulence, strong vertical currents, and icing. The extent of the turbulence is relative to the height of the ground, speed of the wind, and instability of the atmosphere. With adequate moisture, lenticular clouds will form at the top of each wave.

268
Q
The symbol ($) at the end of the following METAR indicates that  
METAR KFSM 131756Z AUTO 00000KT M1/4SM R25/0600V1000FT -RA FG VV004 06/05 A2989 RMK  A02 $.

A. the latest information is broadcast on the voice portion of a local NAVAID at KFSM.
B. the latest information is transmitted over a discrete VHF frequency at KFSM.
C. maintenance is needed on the system.

A

C. maintenance is needed on the system.
Answer (C) is correct. (AC 00-45H Chap 3) ( ? )
A maintenance indicator (dollar) sign, $, is included when an automated weather reporting station detects that maintenance is needed on the system.

269
Q

If the winds aloft are blowing across the front,
A. the front can be expected to move with the upper winds.
B. visibility around the front will be poor due to precipitation.
C. the winds aloft can be expected to turn at the frontal boundary.

A

A. the front can be expected to move with the upper winds.
Answer (A) is correct. (AC 00-6B Chap 10) ( ? )
Systems tend to move with the upper winds. When winds aloft blow across a front, the front tends to move with the wind. When winds blow parallel to the front, little or no movement will occur.

270
Q

Flight in haze
A. is often hazardous when landing into the sun.
B. does not restrict visibility forward or below the aircraft.
C. after sunrise restricts visibility overhead.

A

A. is often hazardous when landing into the sun.
Answer (A) is correct. (AC 00-6B Chap 16) ( ? )
Landing an aircraft into the sun is often hazardous if haze is present.

271
Q

Which condition would INITIALLY cause the indicated airspeed and pitch to increase and the sink rate to decrease?
A. Sudden increase in a headwind component.
B. Tailwind which suddenly increases in velocity.
C. Sudden decrease in a headwind component.

A

A. Sudden increase in a headwind component.
Answer (A) is correct. (AC 00-54) ( ? )
A sudden increase in a headwind component, i.e., a tailwind shearing to a headwind, is initially indicated by an increase in airspeed, pitch, and altitude. A sudden increase in a headwind component will increase aircraft performance.

272
Q

If severe turbulence is encountered, which procedure is recommended?
A. Maintain a constant altitude.
B. Maintain constant airspeed and altitude.
C. Maintain a constant attitude.

A

C. Maintain a constant attitude.
Answer (C) is correct. (FAA-H-8083-25B Chap 5) ( ? )
When severe turbulence is encountered, as in a thunderstorm, it is almost impossible to hold a constant altitude. Maneuvering in an attempt to do so greatly increases stress on the aircraft. Stresses will be least if the aircraft is held in a constant attitude and allowed to ride the waves of updrafts and downdrafts.

273
Q

The following weather may be conducive to severe inflight icing
A. droplets that splash or splatter on impact at temperatures below 0°C ambient temperature.
B. droplets that splash or splatter on impact at temperatures below 10°C and above 1°C ambient temperature.
C. drops that impact at temperatures below -20°C ambient air temperature.

A

A. droplets that splash or splatter on impact at temperatures below 0°C ambient temperature.
Answer (A) is correct. (AC 91-74B) ( ? )
When water droplets splash or splatter on impact at temperatures at or below 0°C, severe icing is possible.

274
Q

The following weather may be conducive to severe inflight icing
A. visible rain with temperature below 10°C.
B. visible moisture with temperature below 5°C.
C. visible rain with temperature below 0°C.

A

C. visible rain with temperature below 0°C.
Answer (C) is correct. (AC 91-74B) ( ? )
Pilots should be especially alert to for severe icing when there is visible rain and the static air temperature is 0°C or colder.

275
Q

What type of report is listed for Lubbock (KLBB) at 1818Z?
A. A special report concerning very low station pressure.
B. A special METAR weather observation, concerning significant weather changes.
C. An aviation selected special weather report.
See figure 145

A
C.	An aviation selected special weather report.
Answer (C) is correct. (AC 00-45H Chap 3) ( ? )
At 1818Z (or 1818 UTC), KLBB issued a SPECI report, which is an aviation selected special weather report. The SPECI is a nonroutine aviation weather report taken when any of the SPECI criteria have been observed.
276
Q

What is a characteristic of the troposphere?
A. It contains all the moisture of the atmosphere.
B. The average altitude of the top of the troposphere is about 6 miles.
C. There is an overall decrease of temperature with an increase of altitude.

A

C. There is an overall decrease of temperature with an increase of altitude.
Answer (C) is correct. (AC 00-6B Chap 1) ( ? )
The troposphere is the layer from the surface to an average altitude of 7 mi. It is characterized by an overall decrease of temperature with an increase in altitude.

277
Q

An inversion aloft with temperatures below freezing can
A. result in the possibility of freezing rain.
B. produce small hailstones.
C. result in heavy frost.

A

A. result in the possibility of freezing rain.
Answer (A) is correct. (AC 00-6B Chap 14) ( ? )
A condition favorable for rapid accumulation of clear icing is freezing rain below a frontal surface. Rain forms above the frontal surface at temperatures warmer than freezing. It falls through air at temperatures below freezing and becomes supercooled. The supercooled drops freeze on impact with an aircraft surface.

278
Q

Which term applies when the temperature of the air changes by compression or expansion with no heat added or removed?
A. Adiabatic.
B. Katabatic.
C. Advection.

A

A. Adiabatic.
Answer (A) is correct. (AC 00-6B Chap 11) ( ? )
When air expands, it cools; when compressed, it warms. These changes are adiabatic, meaning that no heat is removed from or added to the air.

279
Q

What temperature condition is indicated if precipitation in the form of wet snow occurs during flight?
A. The temperature is above freezing at higher altitudes.
B. The temperature is above freezing at flight altitude.
C. There is an inversion with colder air below.

A

B. The temperature is above freezing at flight altitude.
Answer (B) is correct. (AC 00-6B Chap 14) ( ? )
Falling snow may melt in warmer layers of air. When wet snow is encountered, it indicates that the temperature is above freezing at flight altitude.

280
Q

Where can the maximum hazard zone caused by wind shear associated with a thunderstorm be found?
A. Ahead of the roll cloud or gust front and directly under the anvil cloud.
B. In front of the thunderstorm cell (anvil side) and on the southwest side of the cell.
C. On all sides and directly under the thunderstorm cell.

A

C. On all sides and directly under the thunderstorm cell.
Answer (C) is correct. (AC 00-24C) ( ? )
Hazardous turbulence is present in all thunderstorms. The maximum hazard zone caused by wind shear associated with a thunderstorm is found on all sides and directly under the thunderstorm cell.

281
Q

At an indicated altitude of 9,000 feet with a temperature of 85°F, density altitude is
A. 12,800 feet.
B. 13,900 feet.
C. 11,400 feet.

A

A. 12,800 feet.
Answer (A) is correct. (FAA-H-8083-25B Chap 11) ( ? )
Using your E6B on the flight computer side, find the conversion for Celsius to Fahrenheit at the bottom of the flight computer. The temperature of 85°F is about 29.4°C. Now go to the pressure altitude scale and line up just past the mark for 29°C to 9,000 ft. pressure altitude. Note the density altitude arrow points to about 12,750 feet. The closest answer is 12,800 feet.

282
Q

When advection fog has developed, what may tend to dissipate or lift the fog into low stratus clouds?
A. Surface radiation.
B. Wind stronger than 15 knots.
C. Temperature inversion.

A

B. Wind stronger than 15 knots.
Answer (B) is correct. (AC 00-6B Chap 16) ( ? )
Advection fog forms when moist air moves over colder ground or water. It deepens as wind speed increases up to about 15 kt. Wind stronger than 15 kt. lifts the fog into a layer of low stratus or stratocumulus.

283
Q

The Federal Aviation Administration’s Flight Information Service Data Link (FISDL) provides what products?
A. SPECIs, SIGMETs, NOTAMs, and AIRMETs.
B. METARs, SIGMETs, PIREPs, and AIRMETs.
C. Convective SIGMETs, PIREPs, AWWs, and NOTAMs.

A

B. METARs, SIGMETs, PIREPs, and AIRMETs.
Answer (B) is correct. (AC 00-63A) ( ? )
The FAA’s Flight Information Service Data Link (FISDL) provides, free of charge, METARs, SPECIs, TAFs (and their amendments), SIGMETs, convective SIGMETs, AIRMETs, PIREPs (both urgent and routine), and AWWs (issued by the FAA or NWS).

284
Q

What is indicated about an air mass if the temperature remains unchanged or decreases slightly as altitude is increased?
A. A temperature inversion exists.
B. The air is stable.
C. The air is unstable.

A

B. The air is stable.
Answer (B) is correct. (AC 00-6B Chap 12) ( ? )
If the air temperature remains unchanged or decreases only slightly as altitude is increased, the air mass tends to be stable.

285
Q

Dew point fronts are also called
A. wet lines.
B. point fronts.
C. dry lines.

A

C. dry lines.
Answer (C) is correct. (AC 00-6A Chap 8) ( ? )
During a considerable part of the year, dew point fronts are common in western Texas and New Mexico northward over the plains states. Moist air flowing north from the Gulf of Mexico abuts the dryer and, therefore, slightly denser air flowing from the southwest. Except for moisture differences, there is seldom any significant air mass contrast across this “front;” and therefore, it is commonly called a “dry line.” Nighttime and early morning fog and low-level clouds often prevail on the moist side of the line, while generally clear skies mark the dry side. In spring and early summer over Texas, Oklahoma, and Kansas, and for some distance eastward, the dry line is a favored spawning area for squall lines and tornadoes.

286
Q

Where do the maximum winds associated with the jetstream usually occur?
A. Below the jet core where a long straight stretch of the jetstream is located.
B. In the vicinity of breaks in the tropopause on the polar side of the jet core.
C. On the equatorial side of the jetstream where moisture has formed cirriform clouds.

A

B. In the vicinity of breaks in the tropopause on the polar side of the jet core.
Answer (B) is correct. (AC 00-6A Chap 13) ( ? )
Maximum winds associated with the jet stream occur in the vicinity of breaks between the tropical and polar tropopause. Maximum wind speed in the jet stream is found in the core on the polar side due to very strong temperature gradients.

287
Q

What condition is necessary for the formation of structural icing in flight?
A. Supercooled water drops.
B. Visible water.
C. Water vapor.

A

B. Visible water.
Answer (B) is correct. (AC 91-74B) ( ? )
Two conditions are necessary for structural icing in flight: (1) The aircraft must be flying through visible water, such as rain or cloud droplets, and (2) the temperature at the point where the moisture strikes the aircraft must be 0°C or colder.

288
Q

What type weather change is to be expected in an area where frontolysis is reported?
A. The frontal weather is becoming stronger.
B. The front is dissipating.
C. The front is moving at a faster speed.

A

B. The front is dissipating.
Answer (B) is correct. (AC 00-6A Chap 8) ( ? )
Frontolysis is the process in which the adjacent air masses modify, and as temperature and pressure equalize across a front, the front dissipates.